You are on page 1of 95

TỔNG HỢP CÁC BÀI TOÁN VỀ DÃY SỐ, GIỚI HẠN

TRONG ĐỀ THI HSG CÁC TỈNH, THÀNH PHỐ


NĂM HỌC 2011 – 2012 VÀ MỘT SỐ VẤN ĐỀ LIÊN QUAN
(Lê Phúc Lữ - tổng hợp và giới thiệu)

A – ĐỀ BÀI.

Bài 1. (Quảng Bình, vòng 1)

un1
Cho dãy số un  xác định như sau u1  1,  1  un2011 , n  1, 2, 3,...
un

 u 2011 u 2011 u 2011 


Tính lim  1  2  ...  n  .
 u2 u3 un1 

Bài 2. (Vĩnh Long, vòng 1)

u1  3
Cho dãy số un  xác định bởi 
un1  1 un2  un  4 , n  1, 2, 3,...
 5

a) Chứng minh rằng un  là dãy tăng nhưng không bị chặn trên.

b) Đặt vn   , n  1, 2,3,... . Tính lim vn .


n
1
k 1 u k  3
n 

Bài 3. (Chọn đội tuyển THPT chuyên Bến Tre) Tìm số hạng tổng quát của dãy un  thỏa mãn:

u1  u2  1


un 2  2u  u
un 1.un
 n 1 n

Bài 4. (Bình Định, vòng 1)


u1  2  3
Cho dãy số un  được xác định bởi 
  
un1  3  2 un2  2 6  5 un  3 3  3 2
 

1
n
1
Đặt vn   , n  1, 2, 3,... Tìm lim vn .
k 1 u k  2

Bài 5. (Bình Dương, vòng 2)

1 a 
Cho dãy số  xn  được xác định như sau xn   xn1   , n  2 và a  0, x1  0 .
2  xn1 
Chứng minh rằng dãy đã cho có giới hạn và tìm giới hạn của dãy.

Bài 6. (Chọn đội tuyển THPT chuyên Lương Thế Vinh, Đồng Nai)

 x0  a
Cho hai số thực a và b. Xét dãy số  xn  xác định bởi công thức 
 xn1  1  b. xn ; n  
Tìm điều kiện của a, b để  xn  có giới hạn. Tính giới hạn đó.

Bài 7. (Hà Nam, vòng 2)

Cho dãy số thực (xn) thỏa mãn: x1  , xn1 


1 3 xn
2 xn  1
với mọi n nguyên dương.
6
a. Chứng minh dãy số trên có giới hạn và tính giới hạn đó.

b. Tìm số hạng tổng quát của dãy số trên.

Bài 8. (Hà Nội, vòng 1)

1. Cho dãy số un  xác định bởi: u1 = 1 và un1  un  n với mọi n  1 . Tìm lim
un
.
n  un1

2. Cho dãy số vn  xác định bởi: v1  2015 và vn1  vn2  2 với mọi n 1, 2, 3,...

 2011 .
vn21
Chứng minh rằng lim
n  v 2 .v 2 ...v 2
1 2 n

Bài 9. (Long An, vòng 2)

u1  1

Cho dãy số xác định bởi  3u  4
un1  n , n  1, 2, 3,...
 un  1
Đặt xn  u2 n1 , yn  u2 n .
2
a) Chứng minh dãy  xn  ,  yn  có giới hạn hữu hạn.
b) Chứng minh un  có giới hạn hữu hạn và tìm giới hạn đó

Bài 10. (Phú Thọ, vòng 1)

Cho dãy số u1  4, un1 


1
9
 
un  4  4 1  2un , n  1, 2, 3,...

Tìm công thức số hạng tổng quát của dãy số trên.

Bài 11. (Nam Định, vòng 1)

Xét dãy số un  thỏa mãn u1  1, un1  un (un 1)  2, n  1 .

n
Chứng minh rằng An  uk2  1 1 là số chính phương với mọi n.
k 1

Cho dãy số  xn  được xác định bởi:


Bài 12. (Cần Thơ, vòng 2)

 x1  a

 xn1  3 ln  xn  2011   2011
 2011 2 2 2

Chứng minh rằng dãy số  xn  có giới hạn.

Bài 13. (Quảng Ninh, vòng 2)


Cho dãy  xn  xác định bởi x0  a với a  1; 2 và xn1   2
xn
, n  0,1, 2,... .

Chứng minh dãy có giới hạn và tìm giới hạn đó.

Bài 14. (Vĩnh Phúc, vòng 1)

Giả sử a là số thực dương thỏa 0  a  1 . Lập dãy (an ) như sau


a1  a, an 1  a an , n  1 .
Chứng minh rằng dãy có giới hạn hữu han khi n tiến tới vô cực.

Bài 15. (Nam Định, vòng 2)

 
Với mỗi số thực x kí hiệu  x  là số nguyên lớn nhất không vượt quá x và  x  x  x  . Cho

un  (45  2012 ) n . Chứng minh dãy un  có giới hạn và tìm giới hạn đó.

3
Bài 16. (Đà Nẵng, vòng 2)
xn3  3 xn
Cho dãy số thực  xn  thỏa mãn điều kiện xn1 
3 xn  1
2
với mọi n  * .

a) Tìm công thức tính xn theo x1 và n.


b) Chứng minh rằng dãy số  xn  có giới hạn hữu hạn.
Bài 17. (Hưng Yên, vòng 1)

 x1  a  0
Cho dãy số xác định bởi công thức  2
 xn1  xn  xn , n  1
 n2
1 1
Chứng minh rằng  .
xn  n 2
a  n(n  1)
Bài 18. (Quảng Bình, vòng 2)

Cho hai dãy số dương un  , vn  xác định bởi công thức

u  v  2
 1 1
2

 un vn
un1  2 , vn1  , n  1, 2, 3,...
 4vn 1 1 1 4un21
2
a. Tính u2011  v2011
2
.

b. Tính lim un , lim vn .

Bài 19. (Vĩnh Phúc, vòng 2)


n
Cho dãy các số dương (an ) thỏa mãn: ak  2ak 1  ak 2  0,  a j  1, k  1.
j 1

2
Chứng minh rằng 0  ak  ak 1  , k  1 .
k2

Bài 20. (Vĩnh Long, vòng 2)

Xét phương trình x n  x 2  x  1, n  , n  2

a. Chứng minh rằng với mỗi số tự nhiên n  2 thì phương trình trên có đúng một nghiệm dương
duy nhất. Gọi nghiệm đó là xn .

b. Chứng minh rằng lim xn  1 .


n 

4
Bài 21. (Bến Tre, vòng 1)

Cho phương trình x 2 n  3 x  2  0 trong đó n là số tự nhiên lớn hơn 1.

1. Chứng minh rằng ứng với mỗi n, phương trình có đúng một nghiệm xn   0;1 .
2. Gọi  xn  với n  2, 3, 4,... là dãy số có được theo cách xác định như trên. Chứng minh rằng
dãy số này đơn điệu và bị chặn.

Bài 22. (TP HCM, vòng 2)

Cho dãy un  được xác định bởi công thức


u1  5
4


un1  un4
n  *
 un  8un  8
4 2

Tìm công thức tổng quát của dãy un  .

Bài 23. (Tiền Giang, vòng 2)

2  2un2  4un  4
Cho dãy số un  xác định bởi u0  0, un1  , n  1, 2, 3,...
un

Chứng minh rằng dãy un  có giới hạn và tìm giới hạn đó.

Bài 24. (Chọn đội tuyển Phổ thông năng khiếu TP. HCM)

Cho dãy un  thỏa mãn điều kiện u1  và un1  un2  un với mọi n nguyên dương.
1 2
6 3

5un21  2un2un1  5unun1


3un2  unun 1 (4  un2 )
Tính giới hạn sau lim .
n 

Bài 25. (Hà Tĩnh, vòng 2) Dãy số  xn  với n  1, 2, 3,... bị chặn trên và thỏa mãn điều kiện:

xn 2  xn 1  xn với mọi n  1, 2, 3,...


1 3
4 4

Chứng minh rằng dãy số trên có giới hạn.

5
Bài 26. (Ninh Bình, vòng 2)
n
1
Chứng minh dãy un  xác định bởi công thức un    ln n có giới hạn hữu hạn.
k 1 k

Bài 27. (Hà Nội, vòng 2)

Cho dãy số nguyên dương U n  thoả mãn U1  1, U 2  2, U 4  5 và với mọi n  1 thì

U n1U n1  U n2  a với a 2  1 .

1) Xác định số hạng tổng quát của dãy số trên.


2) Tìm các số tự nhiên n không vượt quá 2012 sao cho U n chia hết cho 10.

Bài 28. (KHTN, vòng 3)

2 1 3
Cho dãy số dương an  thỏa mãn a1  1, a2  , an2  an21  an , n  1, 2, 3,...
3 4 4

Chứng minh rằng an hội tụ và tìm giới hạn của nó.

Bài 29. (Chọn đội tuyển ĐHSP Hà Nội)

2n  3
Cho dãy số an  , n 1 thỏa mãn: a1  1, an  an1 , n  2 và dãy bn  thỏa mãn
2n
n
bn   ai , n  1 . Chứng minh dãy bn  có giới hạn hữu hạn, tìm giới hạn đó.
i 1

Bài 30. (Đại học KHTN Hà Nội, vòng 1)

a1  6, a2  14
Cho dãy số an  xác định như sau 
an 2  6an 1  an  24.(1) , n  1, 2,3,...
n

a
n
1
Tính giới hạn lim .
n 
k 1 k

6
B – LỜI GIẢI CHI TIẾT VÀ NHẬN XÉT

Bài 1. (Quảng Bình, vòng 1)

un1
Cho dãy số un  xác định như sau u1  1,  1  un2011 , n  1, 2, 3,...
un
 u 2011 u 2011 u 2011 
Tính lim  1  2  ...  n  .
 u2 u3 un1 

Lời giải.

Từ công thức xác định dãy, ta có

1 1 u 2011 u 2011 1 1
  n  n   , n  1, 2, 3,...
un un1 un1 un1 un un1

n 
u12011 u22011 un2011 n
uk2011 1  1
   
1 1
Do đó   ...      .
u2 u3 un 1 k 1 uk 1  uk uk 1  u1 un 1
k 1 

Dễ thấy rằng un  0, n nên ta cũng có: un1  un  un2012  un hay dãy đã cho tăng thực sự.

Giả sử dãy không có chặn trên thì nó sẽ có giới hạn, đặt đó là  , rõ ràng   1 .

Chuyển công thức tổng quát của dãy về giới hạn, ta có      2012    0 , mâu thuẫn.

Suy ra dãy đã cho không bị chặn trên hay lim un   .

 u 2011 u 2011 u 2011  1 1 


Từ đó, ta được lim  1  2  ...  n   lim     1 .
 u2 u3 un1  1 un 

Nhận xét.

Bài toán này thuộc dạng quen thuộc với ý tưởng rút gọn tổng dưới dạng sai phân để đưa giới hạn
cần tính về giới hạn của dãy ban đầu. Đề bài ở đây rất thuận lợi vì công thức sai phân đã được
thể hiện khá rõ, chỉ cần lập luận cẩn thận, đầy đủ ở các bước là có thể giải quyết trọn vẹn bài này.

Bài 2. (Vĩnh Long, vòng 1)

u1  3

Cho dãy số un  xác định bởi 
un1  1 un2  un  4 , n  1, 2, 3,...
 5

7
a) Chứng minh rằng un  là dãy tăng nhưng không bị chặn trên.

b) Đặt vn   , n  1, 2,3,... . Tính lim vn .


n
1
k 1 u k  3
n 

Lời giải.
a) Dễ thấy với mọi n  0 thì các số hạng của dãy đều dương.
1 2 1 1
Ta có un1  un 
5
un  un  4  un  un2  4un  4  un  2  0 nên dãy đã cho không
5 5
2

giảm. Hơn nữa, từ u1  3  2 nên un  2, n . Từ đó un1  un  0  un1  un , n hay dãy đã


cho đơn điệu tăng.
Giả sử dãy bị chặn trên thì nó phải có giới hạn, đặt là   3 . Chuyển công thức của dãy qua giới
1
hạn, ta được     2    4    2 , mâu thuẫn.
5
Từ đó suy ra dãy này không bị chặn trên. Ta có đpcm.
 1 1  uk 1  uk
 a 
1 1
b) Giả sử ta có công thức    a
uk  3  uk  b uk 1  b  uk  3 uk  b uk 1  b
Quy đồng và biến đổi, ta được (3a  b)un 1  (a 1)un1un  aun2  (3a  b)un  b 2 .

Để tương ứng với công thức quan hệ xây dựng dãy, ta chọn a  1 thì được quan hệ đơn giản hơn
là (3  b)un1  un2  (3  b)un  b 2 , chọn tiếp b  2 thì được công thức đã cho.

1 1 1
Như thế, ta có   , k . Suy ra
uk  3 uk  2 uk 1  2
n 
n
1 
  
1 1 1 1 1
u    
 uk  2 uk 1  2  u1  2 un 1  2
 1 .
k 1 k 3 k 1  un 1  2

n  1 
 lim 1
1
Do lim un   nên lim    1 . Vậy giới hạn cần tìm là 1.
k 1 uk  3  un1  2 

Nhận xét. Trong bài toán này, ta đã dùng phương pháp hệ số bất định để thử tìm một quan hệ có
dạng sai phân giữa các biểu thức liên quan nhằm rút tổng cần tính để tìm giới hạn.
Bài 3. (Chọn đội tuyển THPT chuyên Bến Tre) Tìm số hạng tổng quát của dãy un  thỏa mãn:

u1  u2  1


 
un 1.un
 2un1  un
u n  2

8
Lời giải.

Bài này có thể đổi điều kiện của các số hạng đầu để không rơi vào trường hợp đặc biệt. Ta xét
bài toán tổng quát hơn là:

Tìm số hạng tổng quát của dãy số un  thỏa mãn:

u1  a, u2  b, 2a  b  0


un 2  2u  u , n  1, 2,3,...
un 1un
 n 1 n

2u  u
 n1 n   . Đặt yn  , n  1, 2, 3,...
1 2 1 1
Từ công thức xác định dãy, ta có
un1 un 1un un un1 un

Ta có yn1  2 yn  yn 1 , n . Xét phương trình đặc trưng t 2  2  t  t 2  t  2  0  t  1  t  2 .

Công thức tổng quát của dãy có dạng: yn  r  (2)n  s, n  1, 2,3,...

  a b
2r  s  a r  6ab
1
So sánh với hai số hạng đầu của dãy, ta có:  
 4r  s  1  s  a  2b
 b  3ab

Từ đây thay vào suy ra công thức tổng quát của dãy ban đầu là

xn   , n
a b a  2b
1 6ab
(2)n  (a  b)(2) n  2(a  2b)
6ab 3ab

Trong bài toán ban đầu, nếu thay a  b  1 , ta có công thức tổng quát của dãy là xn  1, n .

Nhận xét.

Trong bài toán trên, ta không nhắc đến điều kiện của a, b để dãy xác định với mọi n.

Điều kiện đó chính là 2 xn 1  xn  0, n hay

  0, n
12ab 6ab
(a  b)(2)  2(a  2b) (a  b)(2) n  2(a  2b)
n 1

Ngoài điều kiện ab  0 suy ra từ đó, ta còn cần có (a  b)  2   2(a  2b)  0, n và


n

9
2(a  b)(2) n  4( a  2b)  ( a  b)( 2) n 1  2( a  2b)  4a( 2) n  2 a  4b  0

Đây chính là hai điều kiện của các số hạng đầu để dãy đã cho luôn xác định.

Ngoài ra, còn một bài toán có giả thiết tương tự như trên nhưng yêu cầu khác:

xn xn1
Cho dãy số  xn  thỏa mãn xn2  , n  * . Tìm điều kiện của x1 , x2 để dãy số trên có
2 xn  xn1
vô hạn số nguyên.

Lời giải.

Đặt x1  a, x2  b, ab  0 . Trước hết, dãy đã cho phải có tất cả các số hạng khác 0.

1 2 1 1
Ta có   , n  *  yn 2  2 yn1  yn , n  * với yn  , n  1 .
xn2 xn 1 xn xn

Phương trình đặc trưng của dãy này có nghiệm kép t  1 nên công thức tổng quát của nó có dạng
1 1
yn  rn  s với r , s được xác định theo y1  , y2  .
a b

r  s  1
Ta có: 
a 1 1 2 1
 r   ,s  
 1 b a a b
2r  s 
 b
 a  b  2b  a ab
Do đó yn   n  xn 
 ab 
.
ab (a  b)n  (2a  a )
Ta thấy a, b nhận những giá trị không đổi và muốn dãy đã cho có vô số số nguyên thì cần phải

(a  b)n  (2b  a ) ab với vô số n. Dễ thấy cần có hệ số trước n phải bằng 0 và a  b .


Khi đó xn  a nguyên khác 0. Thử lại thấy thỏa.
Vậy điều kiện để dãy có vô số số nguyên là a  b   \{0} .

Bài 4. (Bình Định, vòng 1)



u1  2  3
Cho dãy số un  được xác định bởi 
  
un1  3  2 un2  2 6  5 un  3 3  3 2
 

n
1
Đặt vn   , n  1, 2, 3,... Tìm lim vn .
k 1 u k  2

10
Lời giải.

Từ công thức xác định dãy, ta thấy rằng

un1     
3  2 un2  2 6  5 un  3 3  3 2

 un1  3   3 2  u   2  3  3  2  u
2
n n  3  3 2 
 un1  3  3 2 u   2  3  u  6 
2
n n

 un1  3  3 2 u  2 u  3 


n n

1 1 1
  
un1  3 3 2 u n 
 2 un  3 
1 1 1 1 1 1
     
un1  3 un  3 un  2 un  2 un  3 un1  3

n n  
  
1 1 1   1  1
Do đó, vn    .
k 1 u k  2  un  3 un1  3  u1  3 un 1  3
k 1 

Từ đẳng thức un1  3     


3  2 un  2 un  3 và u1  3  2  3 , bằng quy nạp,

ta chứng minh được un  3 , n .

1 1 1
Từ đẳng thức    0 (*) , ta cũng có được un  un1 , n hay dãy đã
un  2 un  3 un 1  3
cho tăng thực sự.

Giả sử dãy đã cho bị chặn trên thì nó có giới hạn, đặt là   3  2 .

1 1 1
Chuyển đẳng thức (*) qua giới hạn, ta được    0 , vô lí.
  2  3  3

Từ đó suy ra dãy này không bị chặn trên hay lim un   .

n  1 
 lim 
1 1   2 .
Do đó, lim vn  lim  
k 1 uk  2  u1  3 un1  3  2

2
Vậy giới hạn cần tìm là .
2

11
Nhận xét.

Tương tự bài đầu tiên, ở bài toán này, ta cũng cần tìm được công thức liên hệ ở dạng thuận lợi
cho việc rút gọn tổng. Tuy biến đổi ở trên khá rắc rối nhưng mục tiêu vẫn là tìm một biểu thức có
dạng như sau

 1 1 
 a 
1
 
un  2  un  b un1  b 

Ta có thể biến đổi rồi đồng nhất hệ số, để tránh các căn thức rắc rối, ta có thể tổng quát nó thành
2  x, 3  y rồi xử lí cho đơn giản hơn.

Một đặc điểm khá thú vị của bài toán này chính là việc chứng minh các số hạng của dãy dương
và dãy đơn điệu tăng không suy ra trực tiếp được từ công thức ban đầu mà phải thông qua các
biến đổi trong quá trình tính toán. Trên thực tế, các dãy số dạng này nói chung luôn có giới hạn
tại vô cực (vì nếu nó có giới hạn là  thì chuyển qua giới hạn trong công thức sai phân, thường
thì ta sẽ thu được mâu thuẫn) nên các quá trình lập luận ở trên có thể nói là thống nhất cho các
dạng tương tự của nó. Một bài tương tự trong kì thi VMO 2009:

1) VMO 2009:
 1
 x1  ,
 2
Cho dãy số ( xn ) xác định như sau  .
 xn21  4 xn1  xn1
 xn  , n  2
 2

Chứng minh rằng dãy số  yn  trong đó yn   2 , n có giới hạn hữu hạn khi n   .
n
1
i 1 xi

2) VMO 2011:
2n n1
Cho dãy số  xn  xác định bởi x1  1, xn   xi , n  2 .
(n 1) 2 i1
Đặt yn  xn 1  xn , n  1, 2,3,... .

Chứng minh dãy  yn  có giới hạn hữu hạn khi n   .

Bài 5. (Bình Dương, vòng 2)

1 a 
Cho dãy số  xn  được xác định như sau xn   xn1   , n  2 và a  0, x1  0 .
2  xn1 
Chứng minh rằng dãy đã cho có giới hạn và tìm giới hạn của dãy.

Lời giải.

12
1 a
Giả sử dãy đã cho có giới hạn, đặt đó là   0 thì ta có          a .
2  

Ta xét biến đổi sau


a xn1  a 
 
2 xn  a  xn1  a 
a
xn1

 a  xn1  a   xn1

 
2
 a  xn1  a
 
 xn1  a 1   
 xn1  xn1

xn1  a x  a
Từ đó suy ra xn  a 
2 xn1
 
xn1  a  xn  a  n1
2 xn1
1
xn1  a  xn1  a .
2

(do xn1  a  xn1 )

1
Lặp lại quá trình này n 1 lần, ta được xn  a  n1
x1  a .
2

Cho n tiến tới vô cực, theo nguyên lí kẹp, ta có lim xn  a  0  lim xn  a .

Vậy giới hạn của dãy đã cho là a và không phụ thuộc vào giá trị của x1 .

Nhận xét.

Bài này có thể giải bằng cách sử dụng hàm số f (t ) liên hệ giữa các số hạng xn , xn1 hoặc dùng
định lí Lagrange. Tuy nhiên, cách đó cần xem xét một số trường hợp nữa và đòi hỏi lập luận
thêm một số trường hợp nữa. Cách giải như trên là đơn giản và nhẹ nhàng hơn cả. Cách tìm ra
giá trị   a cũng rất tự nhiên từ việc giải phương trình sau khi chuyển qua giới hạn.

Bài 6. (Chọn đội tuyển THPT chuyên Lương Thế Vinh, Đồng Nai)

 x0  a
Cho hai số thực a và b. Xét dãy số  xn  xác định bởi công thức 
 xn1  1  b. xn ; n  
Tìm điều kiện của a, b để  xn  có giới hạn. Tính giới hạn đó.

Lời giải.

Xét trường hợp b  1 , ta thấy rằng khi đó xn1  1  xn nên xn  n  a , dãy trong trường hợp
này không có giới hạn.
13
Xét trường hợp b  1 , khi đó xn1  xn  1 nên dãy cũng không có giới hạn.

1 b  1 
Xét trường hợp b  1 , ta có xn1  1  bxn  xn1   bxn   b  xn  .
1 b 1 b 
 1 b 

1  1  n a  ab 1 a  ab 1 1
Suy ra xn   b n  x0    b  xn  b n  , n .
1 b  1 b  1 b 1 b 1 b

1
Dễ thấy rằng nếu b  1 thì lim b n  0 và giới hạn của dãy này là lim xn  .
1 b

a  ab 1
Nếu b  1 thì nếu  0  a  ab  1 thì dãy đã cho không đổi, còn nếu ngược lại thì
1 b
dãy này tiến tới vô cực, tức là nó không có giới hạn hữu hạn.

1
Vậy dãy đã cho hội tụ khi và chỉ khi b  1 hoặc b  1, a  .
1 b

Nhận xét.

Dãy số xác định như trên thay vì biến đổi từng bước như trên, ta hoàn toàn có thể dùng công
thức tổng quát cho dãy truy hồi tuyến tính cấp 1 để xử lí. Các điều kiện về giới hạn của dãy cũng
dễ dàng suy ra được từ các kết quả đó.

Bài 7. (Hà Nam, vòng 2)

Cho dãy số thực (xn) thỏa mãn: x1  , xn1 


1 3 xn
2 xn  1
với mọi n nguyên dương.
6
a. Chứng minh dãy số trên có giới hạn và tính giới hạn đó.

b. Tìm số hạng tổng quát của dãy số trên.

Lời giải.

a. Dễ dàng thấy rằng xn  0, n .

3t 3
Xét hàm số f (t )  , t  0  f  (t )   0 nên đây là hàm đồng biến.
2t  1 (2t  1)2


 x1  1
Dãy số đã cho có thể viết dưới dạng  6

 xn1  f ( xn ), n  1, 2, 3,...

14
3
Do x2   x1 nên x3  f ( x2 )  f ( x1 )  x2 nên bằng quy nạp, ta chứng minh được dãy này
8
3 3 xn 3 3 3
tăng, đồng thời xn1      0 nên xn1  , n .
2 2 xn 1 2 2 xn 1 2

3
Dãy này tăng và bị chặn trên nên có giới hạn, đặt là  thì    2 2  2    0    1 .
2 1

1
Tuy nhiên, do   nên   1 và đây chính là giới hạn cần tìm.
6

b. Do mọi số hạng của dãy đều dương nên ta có thể biến đổi như sau:

1 2 xn  1 2 1 1
   , n  3 yn1  2  yn , n trong đó yn  , n .
xn1 3xn 3 3 xn xn

1 1 1
Với y1   6 , ta tiếp tục biến đổi 3 yn1 1  yn 1  yn 1 1   yn 1  n  y1 1 .
x1 3 3

5 3n1
Suy ra yn1  n 1 hay xn  , n  1, 2, 3,... Đây chính là công thức tổng quát cần tìm.
3 5  3n1

Nhận xét.

Về mặt tìm giới hạn thì dãy số ra trong trường hợp khá chuẩn mực nên có thể tìm được dễ dàng,
ta cũng có thể nhẩm trước rồi trừ vào công thức xác định để đưa về dãy kẹp. Ở bài toán xác định
au  b
công thức tổng quát, thực ra đây là trường hợp đặc biệt của dãy phân tuyến tính un1  n .
cun  d
un1  aun  bvn
Dãy số dạng này được xử lí bằng cách đưa về hệ hai dãy tuyến tính là  .
vn1  cun  dvn
Tuy nhiên, bài toán ở đây đưa ra ở dạng tương đối đặc biệt nên có thể dùng các biến đổi thông
thường để giải quyết.

Bài 8. (Hà Nội, vòng 1)

1. Cho dãy số un  xác định bởi: u1 = 1 và un1  un  n với mọi n  1 . Tìm lim
un
.
n  un1

2. Cho dãy số vn  xác định bởi: v1  2015 và vn1  vn2  2 với mọi n 1, 2, 3,...

 2011 .
vn21
Chứng minh rằng lim
n  v 2 .v 2 ...v 2
1 2 n
15
Lời giải.
n n n
n(n  1) n2  n  2
1. Từ công thức xác định dãy, ta có  ui1   ui   i  un1  u1   .
i 1 i 1 i 1 2 2
(n 1)2  (n 1)  2 n2  n  2
Suy ra un   .
2 2
u n2  n  2
Do đó, lim n  lim 2  1.
n u n n  n  2
n1

Vậy giới hạn cần tìm là 1.

1
2. Vì v1  2015  2 nên ta có thể đặt v1  a  , a  1 . Ta có
a
 1
v2  v  2   a    2  a 2  2
2
1
 a
2
1
a
1
Bằng quy nạp, ta chứng minh được rằng vn1  a 2  2n , n . Ta xét tích
n

 vi    a 2  2i1    a  2i1     a    a  2n
 i1   1   1  n  2i1 1   1   2n 
1 1


      .
n n
1 1
i 1    a   a  i 1  a   a  
a a
i 1 a a
 1  n 1 
 a    a 2  2n 
2 2

Do đó, 2 n21 2  
a  a  , suy ra lim vn 1   a  1    a  1   4  2011 .
   
2 2 2 2
v
 2n 1  v12 .v22 ...vn2  a  a
 a  2n 
2
v1 .v2 ...vn
 a 
Vậy ta có đpcm.
Nhận xét.
Câu 2 của bài toán trên đã từng xuất hiện từ trước khá nhiều, chẳng hạn trong đề Olympic Sinh
viên 2005 (số 2011 ở trên được thay bằng 2005) hoặc trên tạp chí THTT. Trên thực tế, giá trị
2011 có thể thay bằng một đại lượng a bất kì thỏa mãn a  2 bởi vì dãy số có dạng như trên là
một trong các dạng đặc biệt của các dãy phi tuyến tính có thể tìm được công thức tổng quát được.
Tuy việc tìm giới hạn cũng có thể giải theo nhiều cách khác nhưng cách dùng công thức thế này
cho ta nhiều biến đổi đẹp và cơ bản.
Bài 9. (Long An, vòng 2)
u1  1

Cho dãy số xác định bởi  3u  4
un1  n , n  1, 2, 3,...
 un  1
Đặt xn  u2 n1 , yn  u2 n , n   .

16
a) Chứng minh dãy  xn  ,  yn  có giới hạn hữu hạn.
b) Chứng minh un  có giới hạn hữu hạn và tìm giới hạn đó

Lời giải.

3x  4
a) Dễ thấy rằng un  0, n . Xét hàm số f ( x)  ,x0.
x 1

1
Ta có f ( x )   0, x  0 nên đây là hàm nghịch biến. Do đó, suy ra g ( x )  f  f ( x)
( x 1) 2
là hàm đồng biến trên 0; .

Các dãy số đã cho có thể viết lại là

7
u1  1, un1  f un  , n  1 , x1  1, xn1  g ( xn ), n  1 và y1  , yn1  g  yn  , n  1
2

7 29 123
Ta có u2  , u3   u1 , u4   u2 nên dựa vào tính đồng biến của hàm g ( x) , ta có thể
2 9 38
chứng minh được rằng  xn  đồng biến và  yn  nghịch biến.

3un  4 un
Ta thấy rằng un1  4  4   0 nên dãy này bị chặn trên bởi 4 hay dãy  xn 
un  1 un  1
cũng bị chặn trên bởi 4. Suy ra dãy  xn  có giới hạn hữu hạn.

Tương tự, dãy  yn  giảm và bị chặn dưới bởi 0 nên cũng có giới hạn.

Ta có đpcm.

 x2 n 1  f  x2 n 
b) Giả sử lim xn  a, lim yn  b thì do  nên chuyển về giới hạn, ta có hệ
 x2 n 2  f  x2 n 1 


a  3b  4
a  f (b)  b 1 3b  4 3a  4 a b
phương trình sau    a b    .
b  f (a ) b  3a  4 b 1 a 1 (a  1)(b 1)
 a 1

Dễ thấy ngoài hệ thức a  b được suy ra từ hệ trên, không còn quan hệ nào nữa bởi vì
(a 1)(b  1)  1 là vô nghiệm do a, b  0 .

17
3a  4
Với a  b , ta có a   a 2  a  3a  4  a 2  2a  4  0  a  1  5 do a  0 .
a 1

Do đó, hai dãy con  xn  ,  yn  của dãy un  cùng hội tụ về một điểm nên dãy đã cho cũng hội tụ
và giới hạn cần tìm là lim un  1  5 .

Nhận xét.
Bài toán thực ra có thể yêu cầu trực tiếp giới hạn của dãy nhưng dùng thêm hai dãy con như trên
là một gợi ý để việc lập luận có thể dễ dàng hơn cho các bạn mới tiếp xúc với dạng toán tìm giới
hạn thế này.

Bài 10. (Phú Thọ, vòng 1)

Cho dãy số u1  4, un1 


1
9
 
un  4  4 1  2un , n  1, 2, 3,...

Tìm công thức số hạng tổng quát của dãy số trên.

Lời giải.

Từ giả thiết, ta có

9un 1  un  4  4 1  2un  18un1  2un  8  8 1  2un


 18un1  9  (2un  1)  16  8 1  2un

 
2
 9 2un 1  1  2u n  1  4

 3 2un1  1  2un  1  4, n  1, 2, 3,...

vn2 1
Đặt 2un  1  vn , n  un  thì ta có dãy mới tương ứng là
2
v1  3
 .
3vn 1  vn  4, n  1, 2, 3,...
1
Từ công thức xác định dãy này, ta có 3vn 1  6  vn  2  vn1  2  vn  2 , n  1, 2,3,...
3

1  1  
2
1
Suy ra vn  n1  2 , do đó: un   n1  2 1, n  1, 2,3,...
3 2  3  

Đây là công thức tổng quát cần tìm.


18
Bài 11. (Nam Định, vòng 1)

Xét dãy số un  thỏa mãn u1  1, un1  un (un 1)  2, n  1 .

n
Chứng minh rằng An  uk2  1 1 là số chính phương với mọi n.
k 1

Lời giải.
n
Ta sẽ chứng minh bằng quy nạp rằng An  uk2  1 1  un 1 1 , n .
2

k 1

Thật vậy,

- Với n  1 , ta tính được u2  1 nên ta có A1  u12  1 1  u2 1   2 1 , đúng.


2 2

m
- Giả sử khẳng định đúng với n  m  1 , tức là Am  uk2  1 1  um1 1 .
2

k 1

m 1  m 
Ta có Am1  uk2 1 1  uk2 1 um2 1  1 1   Am  1um2 1  1 1 .
k 1
 k 1 

Do đó Am1   Am 1um2 1  1 1  um1 1  1 um2 1 1 1  um2 1  um 1  um2 2 .
2 2

 

Suy ra khẳng định cũng đúng với n  m  1 nên theo nguyên lí quy nạp thì nó đúng với mọi n .
n
Vậy An  uk2  1 1 là số chính phương với mọi n. Ta có đpcm.
k 1

Nhận xét.

Dãy số đã cho không thể tìm được công thức tổng quát nên ta cần phải thông qua các số hạng cụ
thể để có thể dự đoán và chứng min h được quy nạp như thế.

Thật vậy, ta có thể tính được các số hạng của dãy là :

u1  1, u2  2, u3  4, u4  14, u5  184, u6  33674,...

Tương ứng, ta cũng có A1  12 , A2  32 , A3  132 , A4  1832 .

Từ đây dễ dàng dự đoán được điều cần chứng minh ở đây, nhiều bài toán về dãy số nguyên khác
cũng được dự đoán và giải theo cách này.
19
Cho dãy số  xn  được xác định bởi
Bài 12. (Cần Thơ, vòng 2)

 x1  a

 xn1  3 ln  xn  2011   2011
 2011 2 2 2

Chứng minh rằng dãy số  xn  có giới hạn.

Lời giải.

Xét hàm số tương ứng f ( x)  ln  x 2  20112   20112 , x   .


2011
3

 x1  a
Dãy số đã cho chính là 
 xn1  f  xn  , n  1, 2,3,...

1 2  2011x 1
Ta có f ( x)    2  .
2011 2x
3 x  2011 3 x  20112 3
2 2

Xét hàm số g ( x )  f ( x )  x  g ( x )  f ( x)  1  0 nên phương trình g ( x)  0 có không quá

một nghiệm. Ta lại có g (0)  f (0)  ln 20112  20112  0 và g (20112 )  0 nên phương
2011

trình g ( x)  0 có ít nhất một nghiệm do đây là hàm liên tục.


3

Từ đây suy ra phương trình g ( x)  0 có đúng một nghiệm thực.

Gọi a là nghiệm của phương trình g ( x )  0  f (a )  a .

Áp dụng dụng định lý Lagrange cho x, y thuộc  , do hàm f ( x) liên tục trên  nên tồn tại

z  ( x, y ) sao cho: f ( x )  f ( y )  f ( z )( x  y ) , mà f ( z )  , z nên suy ra


1
3
1
f ( x )  f ( y )  x  y với mọi x, y thuộc  .
3
1
Ta có xn1  a  f ( xn )  f (a )  xn  a  ...    x1  a .
n
1
3 3

 1 n 
Dễ thấy rằng lim   x1  a   0 nên theo nguyên lí kẹp, ta có lim xn  a  0 .
n 
 3   n 

Vậy dãy đã có giới hạn hữu hạn. Ta có đpcm.

20
Nhận xét.

Bài toán này được xây dựng trên nguyên lí ánh xạ co với dãy số có công thức truy hồi dạng
xn1  f ( xn ), n  1, 2,3,... và f ( x ) là hàm khả vi thỏa mãn f ( x )  q  1 với q là một số thực
dương nào đấy. Bài toán này được giải theo ý tưởng như trên và nói chung hầu như các bài có
giả thiết thỏa mãn yêu cầu đó đều chứng minh được tồn tại giới hạn theo cùng một cách.

Một bài toán có nội dung tương tự xuất hiện trong đề dự bị VMO 2008 là :

Cho số thực a và dãy số thực {xn } xác định bởi:

x1  a, xn 1  ln(3  cos xn  sin xn )  2008 với mọi n  0,1, 2,...

Chứng minh rằng dãy số  xn  có giới hạn hữu hạn khi n tiến đến dương vô cùng.

Bài 13. (Quảng Ninh, vòng 2)

Cho dãy  xn  xác định bởi x0  a với a  1; 2 và xn1    2


xn
, n  0,1, 2,... .

Chứng minh dãy có giới hạn và tìm giới hạn đó.

Lời giải.

Đặt f ( x)  ( 2) x thì dãy số có dạng x0  2 và xn 1  f ( xn ) .

Ta thấy f ( x) là hàm số tăng và x1   2   2  2  x0 .


x1 1

Từ đó, do f ( x) là hàm số tăng nên ta có x2  f ( x1 )  f ( x0 )  x1 , x3  f ( x2 )  f ( x1 )  x2 ,...

Suy ra {xn } là dãy số tăng. Tiếp theo, ta chứng minh bằng quy nạp rằng xn  2 với mọi n.

 2   2
Thật vậy, điều này đúng với n  0 .

Giả sử ta đã có xk  2 thì rõ ràng xk 1   2.


xk 2

Theo nguyên lý quy nạp, ta có xn  2 với mọi n.

Do đó, dãy {xn } tăng và bị chặn trên bởi 2 nên dãy có giới hạn hữu hạn.

Gọi a là giới hạn đó thì chuyển đẳng thức truy hồi xn 1   2


xn

 2  . Ngoài ra ta cũng có a  2 .
sang giới hạn, ta được

a
a

21
Xét phương trình x   2 ln x
 ln( 2) . Khảo sát hàm số y 
ln x
x
ta thấy rằng phương
x x
trình trên chỉ có 1 nghiệm bé hơn e và một nghiệm lớn hơn e .

Vì 2 là một nghiệm của phương trình nên rõ ràng chỉ có một nghiệm duy nhất của phương trình
thoả mãn điều kiện không vượt quá 2. Từ đó suy ra a  2 .

Vậy giới hạn của  xn  khi n dần đến vô cùng là 2.

Nhận xét.

Các dãy số có hàm số tương ứng dạng f ( x )  a x , a  0 cũng xuất hiện khá nhiều. Trong bài
toán trên, chúng ta có thể tìm được các giá trị của x0 để dãy đã cho có giới hạn.

Trường hợp f ( x ) là hàm đơn điệu giảm, dễ dàng thấy rằng f  f ( x ) là hàm đơn điệu tăng. Khi
đó, dựa theo kết quả phần trên thì từ việc so sánh f  f ( x ) với x, ta sẽ xác định được tính tăng
giảm của hai dãy con  x2n  và  x2 n 1  .

Hai dãy này đơn điệu ngược chiều nhau vì x2 n  f  x2 n1  , x2 n1  f  x2 n  ,... Trong một số
trường hợp, hàm số đã cho không đơn điệu trên cả tập xác định mà chỉ đơn điệu trên miền giá trị
mà các số hạng của dãy nhận được. Ta cần xác định miền đó càng hẹp càng tốt để trên đó, hàm
số đã cho đơn điệu và áp dụng phương pháp đánh giá này trên dãy số đã cho. Tuy nhiên, thay vì
dùng cách xét dãy con như thế, ta có thể thay thế bằng cách dùng định lí Lagrange để giúp đơn
giản hóa các bước lập luận và các bước giải sẽ nhẹ nhàng hơn.

Bài 14. (Vĩnh Phúc, vòng 1)

Giả sử a là số thực dương thỏa 0  a  1 . Lập dãy (an ) như sau a1  a, an 1  a an , n  1 .


Chứng minh rằng dãy có giới hạn hữu han khi n tiến tới vô cực.

Lời giải.

Ta thấy rằng 0  an  1, n .
Xét hàm số f ( x)  a x , x  0;1 là hàm liên tục trên 0;1 với f ( x )  a x ln a  0 nên là hàm
nghịch biến.

Dãy số đã cho có thể viết lại là a1  a, an 1  f (an ), n  1, 2,3,...

1  1 
Ta có f ( x )  a x ln a  a ln a  a ln a  a ln  a ln 1  1 .
a  a 
22
1 
Theo bất đẳng thức quen thuộc là ln t  1  t , t  0 , ta được f ( x )  a  1  1 a  1 .
 a 

Sử dụng định lý Lagrange cho hàm f ( x ) trên miền  a; b , 0  a  b , ta thấy rằng tồn tại số thực
c   a; b sao cho
f ( x )  f ( y )  f (c ) x  y  1 a  x  y .
Xét phương trình a x  x tương ứng với hàm số g ( x )  a x  x .

Dễ thấy hàm này có g ( x )  a x ln a 1  0 nên nghịch biến và g (0)  1  0, g (1)  a 1  0 nên


theo tính chất hàm liên tục thì tồn tại x0 sao cho g ( x0 )  0 hay phương trình a x  x có nghiệm
duy nhất trên miền (0;1) là x  x0 . Suy ra f  x0   x0 .

Từ đó, ta có f (an )  f ( x0 )  (1 a ) an  x0  an1  x0  (1 a) an  x0 .

Áp dụng nhiều lần đành giá này, ta được an  x0  (1 a)n a1  x0 .

Từ đó, theo nguyên lí kẹp thì dãy này hội tụ và và giới hạn của dãy là nghiệm x  x0 duy nhất
của phương trình x  a x .

Nhận xét.

Như vậy các dãy số có dạng xn 1  f ( xn ) và hàm f ( x ) đơn điệu gần như đã được giải quyết
hoàn toàn. Tuy nhiên, trong các trường hợp khác, dãy này không đơn điệu thì việc biện luận
phức tạp hơn nhiều. Khi đó, ta sẽ dùng kết quả sau:

Nếu f ( x) là một hàm số thỏa điều kiện tồn tại số thực 0  q  1 sao cho f ( x )  f ( y )  q. x  y
với mọi x, y thuộc tập xác định thì dãy số  xn  xác định bởi x0  a  I , xn 1  f ( xn ) hội tụ.
Giới hạn của dãy số là nghiệm duy nhất trên I của phương trình x  f ( x ) .

Chứng minh.
Theo tính chất của dãy, với mọi n  m thì

xn  xm  f ( xn 1 )  f ( xm 1 )  q. xn 1  xm1  ...  q m . xn  m  x0 (*)

Từ đây xn  x0  xn  xn 1  xn 1  xn 2  ...  x1  x0  (q n 1  q n  2  ...  q  1) x1  x0 ,

suy ra  xn  bị chặn. Xét một số thực   0 .

Từ (*), do q  1 và xn  m  x0 bị chặn nên ta suy ra tồn tại N sao cho q n xn  m  x0   . Theo


định nghĩa giới hạn thì suy ra dãy này hội tụ.
23
Trong nhiều trường hợp, việc chứng minh sự tồn tại của số thực q  0;1 là điều không đơn giản
và ta thực hiện được điều đó qua cách xét đạo hàm của hàm số này. Nếu như 0  f  ( x)  q  1
thì với mọi x, y  ; x  y , theo định lí Lagrange thì tồn tại c   y; x  sao cho

f ( x)  f ( y ) f ( x)  f ( y)
 f  (c )   q  f ( x)  f ( y )  q x  y .
x y x y

Từ đây ta tiến hành khảo sát dãy đã cho và dùng nguyên lí kẹp để suy ra giới hạn.

Một bài toán tương tự (thuộc dạng cụ thể hóa) đã từng xuất hiện trên tạp chí THTT:

 1 n
a

Xét dãy (an ) như sau a1  1, an1    , n  1 .


 27 
Chứng minh rằng dãy có giới hạn hữu hạn khi n tiến tới vô cực.

Bài 15. (Nam Định, vòng 2)

 
Với mỗi số thực x kí hiệu  x  là số nguyên lớn nhất không vượt quá x và  x  x  x  . Cho

un  (45  2012 ) n . Chứng minh dãy un  có giới hạn và tìm giới hạn đó.

Lời giải.

Trước hết, ta sẽ chứng minh rằng với mọi số nguyên dương n thì biểu thức sau nhận giá trị

   
n n
nguyên An  45  2012  45  2012 .

Thật vậy, theo khai triển nhị thức Newton thì


n n

       
n n i i
An  45  2012  45  2012   Cni 45ni 2012   Cni (1)i 45ni 2012
i 0 i 0

   
2i 2 i 1
 
02 in
Cn2 i 45n2 i 2012  
02 i 1n
Cn2 i 1 45n2 i1 2012 

   
2i 2 i 1
 
02 i n
Cn2 i 45n2 i 2012  
02 in
Cn2 i 1 (1)2 i 1 45n2 i1 2012

2 
02 i n
2i
C 45
n
n2 i
2012 i

là số nguyên với mọi n.

 
n
Hơn nữa, ta thấy rằng 0  45  2012  1 và lim 45  2012  0 nên
24
 n
   
n
An 1  45  2012  An   45  2012   An 1 .
 

Do đó

45     45   n
  
n n
2012 2012   45  2012 
 

   
n n
 An  45  2012  ( An 1)  1  45  2012

  45    cũng hội tụ và


n n
Do 45  2012 hội tụ nên 2012

    1 lim 45  
n n
lim 45  2012 2012 1

Vậy giới hạn cần tìm là 1.


Nhận xét.

   trong đó 0  a 
n
Bài toán có thể tổng quát lên thành dạng tìm giới hạn của a  b b 1 .

Cách xử lí vẫn tương tự nhưng trong cả hai trường hợp a 2  b hay a 2  b thì khi khai triển, các
đại lượng chứa căn vẫn bị triệt tiêu và giới hạn luôn là 1.
Bài 16. (Đà Nẵng, vòng 2)
xn3  3 xn
Cho dãy số thực  xn  thỏa mãn điều kiện xn1 
3 xn  1
2
với mọi n  * .

a) Tìm công thức tính xn theo x1 và n.


b) Chứng minh rằng dãy số  xn  có giới hạn hữu hạn.
Lời giải.
a) Ta thấy rằng nếu x1  1 thì xn  1, n và nếu x1  1 thì xn  1, n .

Ta xét trường hợp x1  1  xn  1, n . Từ công thức xác định dãy số, ta có:

xn3  3xn  3xn2  1 ( xn  1)3 xn3  3 xn  3xn2  1 ( xn  1) 3


xn1  1   , xn 1  1  
3xn2  1 3 xn2  1 3xn2  1 3 xn2  1

x 1  xn 1
3
x 1 y 1
Do đó, n1  , n . Đặt yn  n , n  1, 2, 3,...  xn  n
xn1  1  xn  13
xn  1 yn  1

25
x1 1 n
Ta có y1  , yn1  yn3 , n  1, 2, 3,... Bằng quy nạp, ta chứng minh được yn  y13 .
x1  1

3n
 x1 1
  1
 x1  1
n n n
y13  1 ( x1 1)3  ( x1 1)3
Suy ra xn1  n  3n
 n n , n .
y13 1  x 1  ( x1 1)3  ( x1 1)3
 1  1
 x  1
1

Đây là công thức tổng quát cần tìm.


x3  3 x
b) Xét hàm số f ( x)  2 có đạo hàm dương và dãy số đã cho có thể viết lại dưới dạng
3x 1
xn1  f  xn  , n  1, 2,3,...
Với x1  1 thì ta cũng có tương ứng xn  1 nghĩa là dãy đã cho có giới hạn.

2 x 3  2 x 2 x ( x 2 1)
Với x1  1 thì f ( x )  x   . Ta có các trường hợp sau:
3x2  1 3 x2 1
+ Nếu x1  1 thì x2  x1  f ( x)  x  0 nên dãy đã cho giảm. Thật vậy,

Từ x2  x1  f ( x2 )  f ( x1 )  x3  x2 . Bằng quy nạp, ta chứng minh được xk 1  xk , k nên


dãy đã cho giảm. Hơn nữa, bằng quy nạp, ta cũng chứng minh được rằng xn  1, n nên dãy đã
cho có giới hạn hữu hạn.

+ Nếu 0  x1  1 thì x2  x1  0 nên dãy đã cho tăng; dãy này bị chặn nên cũng có giới hạn.

+ Trường hợp 1  x1  0 và x1  1 cũng chứng minh tương tự.

Vậy với mọi giá trị của x1 , ta thấy dãy đã cho luôn có giới hạn.

Nhận xét.

x3  3 x x 3  3ax
Hàm số f ( x)  có thể tổng quát lên bằng f ( x )  với cách giải tương tự, do ta
3 x2 1 3x 2  a
3( x 2  a)2
cũng có f ( x )   0 . Việc xét trước các trường hợp x1  1 để tránh rắc rối là việc
3x 2  a
2

làm cần thiết. Công thức tổng quát tìm được không giúp được nhiều trong việc biện luận giới hạn
dãy số ở đây và trên thực tế thì việc sử dụng tính đơn điệu của dãy số hoàn toàn có thể được
dùng để xử lí trong tình huống này.

26
Bài 17. (Hưng Yên, vòng 1)

 x1  a  0
Cho dãy số xác định bởi công thức  2
 xn1  xn  xn , n  1
 n2
1 1
Chứng minh rằng:  .
xn  n 2
a  n(n  1)

Lời giải.

Bất đẳng thức cần chứng minh tương đương với xn  n 2  a  n(n 1) .

Ta sẽ chứng minh điều này bằng quy nạp. Thật vậy,

- Với n  1 , ta có x1 1  2 a  a  1  2 a , bất đẳng thức này đúng.

- Giả sử khẳng định đúng với n  k  1 , tức là xk  k 2  a  k (k  1) .

xk2
Ta cần chứng minh xk 1  (k 1)  a  (k  1)(k  1)  xk  2  (k  1)2  a  (k 1)(k  2) .
2

k
Ta sẽ chứng minh rằng
 a  k (k 1)  k 2  a  k (k  1)
 
2
 (k  1)2  a (k  1)(k  2)
k
  a (k 1)  k  a  k  1  a (k  2)
 
 
2
 a(k 1)  k  1  2 a (k  1)  (k 1) a 1  0

Do đó, khẳng định đúng với n  k  1 nên theo nguyên lí quy nạp, ta có đpcm.

Nhận xét. Từ chứng minh đánh giá ở trên, ta có thể tìm được các giá trị của a để dãy đã cho có
giới hạn hữu hạn.

Bài 19. (Vĩnh Phúc, vòng 2)


n
Cho dãy các số dương (an ) thỏa mãn: ak  2ak 1  ak 2  0,  a j  1, k  1.
j 1

2
Chứng minh rằng 0  ak  ak 1  , k  1 .
k2
Lời giải.

27
Từ giả thiết, ta có ai  ai 1  ai 1  ai 2 , i  1 .
n
Do a
j 1
j  1 và a j  0, j nên 0  ai  1, i  1 .

Xét số nguyên dương k bất kì, ta có


uk  uk 1  uk 1  uk 2  ...  un1  un
uk  u n
Do đó, uk  un   n  k uk  uk 1   uk  uk 1  .
nk
1
Ta cũng có uk  un  0 1  1 nên uk  uk 1  , n .
nk
1
Điều này đúng với mọi n nên ta có uk  uk 1  lim 0.
n n  k

Do đó, ta đã chứng minh được bất đẳng thức thứ nhất là uk  uk 1  0, k

k k
Ta cũng có 1   ui   j u j  u j 1   kuk , mà uk  0 nên
i 1 i 1
k
k (k  1) 2 2
1   j u j  u j 1   kuk  uk  uk 1  suy ra uk  uk 1   2.
i 1 2 k (k 1) k
Nhận xét. Đây là một trong số ít các bài toán không đưa ra theo hệ thống các kĩ thuật có sẵn mà
đòi hỏi việc tư duy linh hoạt trong việc biến đổi và vận dụng các công thức, tận dụng điểm đặc
trưng của các liên hệ xuất hiện trong bài.

Ta sẽ tìm hiểu bài toán tính giới hạn sau đây để thấy rõ điều đó:

Cho dãy số an  xác định bởi a1  0 và an 1  an 


n
với mọi n  1 . Chứng minh rằng
an
a) an  n với mọi n  2 .
a 
b) Dãy số  n  có giới hạn hữu hạn và tìm giới hạn đó.
n
c) lim  an  n   0.
x 

Lời giải.
a) Ta sẽ chứng minh an  n với mọi n  2 bằng quy nạp. (*)

1
Với n  2 , ta có a2  a1   2 do a1  0 nên (*) đúng.
a1

Giả sử (*) đúng với n  k  2 , tức là ak  k . Ta cần chứng minh (*) cũng đúng với n  k  1 ,
k
tức là ak 1  ak   k 1 .
ak
28
Nhưng bất đẳng thức trên đúng vì nó tương đương với
ak2  (k 1)ak  k  0  (ak 1)(ak  k )  0

Do đó, (*) cũng đúng với n  k  1 . Theo nguyên lí quy nạp, (*) được chứng minh.
 an  an1
b) Ta sẽ chứng minh dãy 
a
  đơn điệu giảm, tức là n  , n  2 .
 n  n n 1
n
Thay an 1  an  vào bất đẳng thức trên, ta có
an

an an  n / a n
  (n  1)an  nan  n  an  n
n n 1
Bất đẳng thức này đúng nên nhận xét trên được chứng minh. Hơn nữa, vì an  n, n  2 nên dãy

 an 
  bị chặn dưới, suy ra nó có giới hạn.
 n 
n n n n
n i i
Ta có an 1  an 
an
, suy ra  ai1   ai  
i 2 i 2 i 2 ai
 an1  a2   .
i 2 ai

n
i n
a a n2
Do i  ai , i  2, n nên 
i 2 ai
 
i 2
1  n  2  an1  a2  n  2  n 1  2 
n 1 n 1 n 1
.

an1 a n2  a n  2 
Ta thấy rằng 1   2  và lim  2    1 nên theo nguyên lí kẹp, giới
n 1 n 1 n 1 
n  n  1 n 1 
 a 
hạn của dãy  n  là 1.
 n 

n an2  an (n  1)  n (an  n)(an 1)


c) Ta có an 1  (n  1)  an  n  1   . Suy ra
an an an
n 
n n
(ai  i )(ai 1) 1
  ai1  (i  1)    an 1  (n  1)  (a2  2)1 
i 2 i 2 ai i 2  ai 
n a 1 a 1 a
Ta có an 1  an   an  1  n 1  n  1  n . Do đó
an an1 an1 an1 an 1
n
ai1 a a (a  2)
0  an 1  (n  1)  (a2  2) (a2  2) 1  1 2
ai an n

 0 nên ta được lim  an  n   0. Ta có đpcm.


i 2

a1 (a2  2)
Cũng theo nguyên lí kẹp, do lim
x  n x 

29
Bài 20. (Vĩnh Long, vòng 2)

Xét phương trình x n  x 2  x  1, n  , n  2

a. Chứng minh rằng với mỗi số tự nhiên n  2 thì phương trình trên có đúng một nghiệm dương
duy nhất. Gọi nghiệm đó là xn .

b. Chứng minh rằng lim xn  1 .


n 

Lời giải.

a) Ta thấy rằng nếu x là nghiệm của phương trình x n  x 2  x  1, n  , n  2 thì x  1 .

Với n  3 cố định, xét hàm số f n ( x )  x n  ( x 2  x  1), x  1 . Ta có

f n( x )  nx n1  (2 x  1)  3 x  (2 x  1)  x 1  0 nên đây là hàm đồng biến nên phương trình
f n ( x)  0 có không quá một nghiệm.

Hơn nữa, f n (1)  2, lim f n ( x)   và hàm số f n ( x ) liên tục trên (0; ) nên phương
x

trình f n ( x)  0 có ít nhất một nghiệm dương.

Từ các điều trên, ta thấy rằng phương trình x n  x 2  x 1 có đúng một nghiệm dương.

b. Ta sẽ chứng minh dãy này giảm. Thật vậy, theo định nghĩa xn thì f n  xn   0, f n1  xn 1   0 ,
tức là xnn  xn2  xn  1  xnn1  xn3  xn2  xn  xnn 1   xn2  xn  1  xn3 1  0 .

Suy ra f n 1 ( xn )  0  f n1  xn1  , mà f n 1 ( x) đồng biến nên xn  xn1 , n .

Do  xn  bị chặn dưới bởi 1 nên nó có giới hạn hữu hạn.

 1 
Ta sẽ chứng minh rằng f n 1    0 . Thật vậy, ta xét n  4 thì có đánh giá sau
 n 

     
n 2
1  1   1  Cn  Cn  1  n  n(n 1)  1  n  n 1  1  1   1  1  1 .
1 2

 n  n n 2n 2  n   n 

 1  1 1
Suy ra f n 1    0  f n  xn   1   xn . Do đó 1  xn  1  .
 n n n

Theo nguyên lí kẹp thì dãy này có giới hạn là 1.


30
Nhận xét.

Các bài toán về biểu diễn nghiệm của một phương trình rồi xác định giới hạn thường khá thú vị
và trong phần chứng minh tồn tại giới hạn, ta luôn sử dụng tính đơn điệu của dãy số để nhận xét,
chú ý f n 1 ( xn1 ) và f n 1 ( xn ) . Trong phần tìm chính xác giới hạn của dãy, để tránh sự nhầm lẫn
và ngộ nhận ở một số trường hợp, ta nên dùng dãy kẹp để chứng minh giới hạn (thường thì đề
bài sẽ cho trước giới hạn này).

Các bài toán tương tự đã xuất hiện trong đề VMO các năm trước :

1) Đề VMO 2002, bảng A :

1 1 1 1
Xét phương trình   ...  2  với n là tham số nguyên dương.
x 1 4 x 1 n x 1 2

a) Chứng minh rằng với mỗi số nguyên dương n thì phương trình nêu trên có duy nhất một
nghiệm lớn hơn 1. Kí hiệu nghiệm đó là xn .

b) Chứng minh dãy số  xn  có giới hạn là 4 khi n   .

2) Đề VMO 2002, bảng B :


1 1 1 1
Xét phương trình    ...   0 với n là tham số nguyên dương.
2 x x 1 x  4 x  n2
a) Chứng minh rằng với mỗi số nguyên dương n thì phương trình nêu trên có duy nhất một
nghiệm lớn hơn 1. Kí hiệu nghiệm đó là xn .

b) Chứng minh dãy số  xn  có giới hạn hữu hạn khi n   .

3) Đề VMO 2007 :

Xét số thực a  2 . Đặt f n ( x )  a10 x n10  x n  x n1  ...  x  1, n  1, 2,3,...

Chứng minh rằng với mỗi giá trị nguyên dương của n thì phương trình f n ( x)  a có đúng một
1
nghiệm xn  0;  và dãy số xn có giới hạn là 1 khi n   .
a

Bài 18. (Quảng Bình, vòng 2)



u  v  2
 1 1
Cho hai dãy số dương un  , vn  xác định bởi 
2
 un vn
un1  2 , vn1  , n  1, 2, 3,...
 4vn 1 1 1 4un21
31
2
a. Tính u2011  v2011
2
.

b. Tính lim un , lim vn .

Lời giải.

a) Ta sẽ chứng minh bằng quy nạp rằng un2  vn2  1, n .

1 1
Thật vậy, với n  1 , ta có u12  v12    1 .
2 2

Giả sử (*) đúng với n  k , tức là uk2  vk2  1 thì theo công thức xác định dãy, ta có

uk  uk 1 4vk21 1 , vk  vk 1 1 uk21  . Do đó, ta có biến đổi sau

uk 1 (4vk 1  1)    vk 1 (1  4uk 1   1  uk 1 (16vk 1  8vk 1  1)  vk 1 (16uk 1  8uk 1  1)  1


2 2 2 2 2 4 2 2 4 2

 (uk21  vk21  1)(16uk21vk21  1)  0  uk21  vk21  1

Suy ra khẳng định cũng đúng với n  k  1 .

Theo nguyên lí quy nạp, ta có un2  vn2  1, n , suy ra u2011


2
 v2011
2
 1.

b) Do un , vn  0 nên ta có: 1 4un21  1  vn1 


vn
 vn . Do đó dãy vn  tăng và bị chặn
1 4un21
trên bởi 1 nên tồn tại lim vn  b .

 4vn1 1  4   1  1 . Ta được un 1  2 n  un .


2  2 2 u
4vn 1  1
Ta cũng có vn  v1  2

2  2 

Dãy số un  giảm và bị chặn trên bởi 0 nên cũng tồn tại lim un  a .

Trong công thức xác định dãy, ta chuyển qua giới hạn: a   a (4b 2  2)  0 .
a
4b  1
2

Nếu a  0 thì b 
2 2
(vô lí do b  ) , suy ra a  0 , tương ứng với b  1 .
2 2
Vậy lim un  0, lim vn  1 .
Nhận xét. Bài toán này có dạng dãy số xác định theo kiểu lượng giác. Nếu đề bài ban đầu không
cho chứng minh quan hệ giữa hai số hạng như câu a thì thực sự là không dễ dàng để tìm được cả
hai giới hạn của dãy đã cho. Trong phần chứng minh ở câu a, ý tưởng quy nạp là rất hiển nhiên !

32
Bài 21. (Bến Tre, vòng 1)

Cho phương trình x 2 n  3 x  2  0 trong đó n là số tự nhiên lớn hơn 1.

1. Chứng minh rằng ứng với mỗi n, phương trình có đúng một nghiệm xn   0;1 .
2. Gọi  xn  với n  2, 3, 4,... là dãy số có được theo cách xác định như trên. Chứng minh rằng
dãy số này đơn điệu và bị chặn.

Lời giải.

1. Với n  2 cố định, ta xét hàm số f n ( x )  x 2 n  3 x  2, x  0;1 .

Ta có f n( x )  2n(2n 1) x 2 n2  0, 0  x  1 nên phương trình f n ( x)  0 có không quá 2 nghiệm.

3
Mặt khác f n( x )  2nx 2 n1  3, f n( x)  0  x  x0  2 n1  0;1 và f n( x0 )  0 nên đây là giá
2n
trị cực tiểu trên miền (0;1) .

Suy ra f n ( x0 )  f n (1)  0 . Từ đó, do tính liên tục của hàm số này và f n (0)  0, f n ( x0 )  0 nên
phương trình f n ( x)  0 còn có nghiệm trong khoảng xn  0; x0   0;1 . Do f n ( x)  0 có không
quá hai nghiệm nên đây là nghiệm duy nhất. Ta có đpcm.

2. Ta có f n ( xn )  0  xnn  3xn  2  0 . Suy ra xnn1  3xn2  2 xn  0 (nhân hai vế cho xn  0) .

 2  2 
n

Ta cũng có f n       0 và f n ( x ) nghịch biến trên khoảng 0; x0  , đồng biến trên khoảng
 3   3 
2 2
(0; x0 ) nên ta có  0; xn  hay xn  , n .
3 3

Do đó, xnn1  3xn  2  3 xn2  2 xn  3xn  2  3 xn2  5 xn  2  (3 xn  2)( xn 1)  0 .

Trong khi đó f n 1 ( xn1 )  0  xnn11  3xn1  2  0 .

Tương tự, xét các khoảng nghịch biến và đồng biến của hàm số f n 1 ( x) thì ta thấy rằng vì
f n 1 ( xn1 )  0, f n 1 ( xn )  0 nên xn   xn1 ,1  xn1  xn , n .

2
Từ đó ta được dãy này giảm và bị chặn dưới bởi . Ta có đpcm.
3

Nhận xét.
33
Bài này tuy cũng thuộc dạng tương tự bài 19 và các bài toán tương tự đã nêu trong phần nhưng
xét nhưng bên cạnh đó vẫn còn một số vấn đề xuất hiện. Rõ ràng hàm số f n ( x ) nêu trong bài
không đơn điệu và phương trình đã cho luôn có hai nghiệm. Vì thế, ta cần phân tích sâu hơn vào
đạo hàm cấp 1, cấp 2 để dựa vào tính biến thiên của dãy mà lập luận. Tuy bài này chỉ yêu cầu
2
chứng minh dãy có giới hạn nhưng ta vẫn có thể tìm được giới hạn đó là bằng dãy kẹp.
3

Bài 22. (TP HCM, vòng 2)

Cho dãy un  được xác định bởi công thức


u1  5
4


un1  un4
n  *
 un4  8un2  8

Tìm công thức tổng quát của dãy un  .

Lời giải.

1 1 u 4  8un2  8 8 8
Đặt  vn , n . Từ công thức xác định dãy, ta có  n 4
 1  2  4 hay
un un1 un u n un
5
vn1  8vn4  8vn2  1, n  1, 2, 3,... và v1 
4


1 2 2 n2

2 n2
Bằng quy nạp, ta sẽ chứng minh rằng vn  2  22 , n  1, 2, 3,... .
2
Thật vậy,
1 1 5

1 222
2  22  2    . 
22
- Với n  1 , ta có v1 
2 2  2 4


1 22 k2

2 k 2
- Giả sử với n  k 1 , ta có vk  2  2 2 . Theo công thức tổng quát của dãy, ta được
2


8 2 2 k2
  48 2  8
2 k 2 4 2
22 k2 2 k 2
vk 1  vk4  8v22  8  2  22  2 2
16


1 22 k1
  
2 k 1 2 2 k 1 2 k 1
 2  22  2  2 22  22  2  1
2


1 2 k1
    1 2k
  
2 k 1 2 2 k 1 2 k 1 2 k 1 2 k 1
 2 2  2 2  2 22  22  2  2 22  22  2  1  22  22
2k

2 2

34
Do đó khẳng định đúng với n  k 1 . Theo nguyên lí quy nạp, khẳng định trên đúng.

2
Vậy un  22 n2 2 2 n2
, n  * .
2 2

Nhận xét.

Các công thức kiểu như trên thường gợi nhớ đến các hàm lượng giác và tương ứng với nó là các
hàm hypebolic mà công thức liên hệ cũng giống như của các hàm lượng giác quen thuộc.

Các bài toán về dạng này không nhiều và các giải chung đều là dự đoán công thức rồi dùng quy
nạp để chứng minh. Các công thức thường có dạng a 1k   với  ,  , a, k được chọn
f (n) f (n)
 2k 1 2

theo các giá trị cho trước, hàm f (n) thì tùy theo hàm số trong hệ thức truy hồi).

Một số bài tương tự:

1) Cho dãy số un  , n  1, 2, 3,... có u1  1, un 1  9un3  3un . Tìm công thức tổng quát của dãy.

2
2) Cho dãy số vn  , n  1, 2, 3,... xác định bởi v1  , vn1  64vn5  40vn3  5vn , n  1, 2, 3,... .
3

Tìm công thức tổng quát của dãy.

Bài 23. (Tiền Giang, vòng 2)

2  2un2  4un  4
Cho dãy số un  xác định bởi u0  0, un1  , n  1, 2, 3,...
un

Chứng minh rằng dãy un  có giới hạn và tìm giới hạn đó.

Lời giải.

2 4 4
Do un1   2   2 , n  1, 2,3,... nên dễ dàng thấy rằng un  0, n .
un un un

2 4 4 2 4 4
Ta cũng có un  2, n nên un1   2  2   2  5.
un un un 2 2 2

Do đó, dãy đã cho bị chặn.

35
2 4 4
Xét hàm số f ( x)   2  2 , 2  x 5.
x x x

Dãy số đã cho có thể viết lại thành u0  0, un1  f un  , n  0,1, 2,...

Ta thấy hàm này nghịch biến trên khoảng xác định nên f  f ( x ) đồng biến.

Xét dãy an  u2 n , bn  u2 n 1 , n  0 .

Nếu u0  u2 thì dãy an  đồng biến và bn  nghịch biến, cả hai đều bị chặn nên có giới hạn.
Tương tự nếu u0  u2 thì cả hai dãy cũng có giới hạn.

Đặt lim an  a, lim bn  b thì từ công thức xác định dãy, chuyển qua giới hạn, ta có


b  2  2  4  4

 ab  2  2  4a  4a
2
a a a2
   a  a 2  b  b2  a  b
 2 4 4 
ab  2  2  4b  4b 2
a   2   2
 b b b

Ta xét phương trình

a 2  2  2  4a  4a 2  (a 2  2) 2  4a 2  4a  2  ( a  2)( a  3 1)( a  3 1)  0


 a  3 1  0

Do đó, hệ này có nghiệm duy nhất là a  b  3 1 .

Vậy hai dãy con của dãy đã cho có cùng giới hạn nên dãy un có giới hạn và lim un  3  1 .

Nhận xét.

Đây lại là một minh họa điển hình về dãy số xác định theo kiểu un1  f un  với f ( x ) là hàm
đơn điệu giảm, các dãy này thường ít khi có giới hạn phụ thuộc vào số hạng đầu.

Một bài tương tự trong đề VMO 2008:

1
Cho dãy số  xn  xác định như sau x1  2, x2  0, xn 2  2 xn  với mọi n  1, 2,3,...
2

Chứng minh dãy  xn  có giới hạn hữu hạn. Tìm giới hạn đó.

36
Bài 24. (Chọn đội tuyển Phổ thông năng khiếu TP. HCM)

Cho dãy un  thỏa mãn điều kiện u1  và un1  un2  un với mọi n nguyên dương.
1 2
6 3

5un21  2un2un1  5unun1


3un2  unun 1 (4  un2 )
Tính giới hạn sau lim .
n 

Lời giải.

1
Trước hết, ta sẽ tìm giới hạn của dãy un  . Bằng quy nạp, ta sẽ chứng minh rằng 0  un  , n .
3
Thật vậy,

- Với n  1 thì khẳng định đúng.

1
- Giả sử khẳng định đúng với n  k 1 thì 0  uk  . Ta có
3

2 1 2 1 1
0  uk 1  uk2  uk     nên khẳng định cũng đúng với n  k 1 .
3 9 3 3 3

Theo nguyên lí quy nạp, khẳng định được chứng minh.

2  2 2
Xét hàm số f ( x )  x 2  x, x   0;  thì f ( x )  2 x   0 nên đây là hàm đồng biến.
3 
 3 3

1
Dãy số đã cho chính là u1  , un 1  f un  , n  1, 2, 3,...
6

1 2 1 5 1
Hơn nữa u2  2
    nên đây là dãy nghịch biến và bị chặn dưới nên có giới hạn.
6 3 6 36 6

2 1
Gọi l là giới hạn của dãy thì l  l 2  l  l  0  l  . Nhưng do dãy này giảm và theo chứng
3 3
1
minh trên thì 0  un  , n nên giới hạn của dãy là 0.
3

un1 2  un1 
Theo công thức xác định dãy, ta có  un  . Do dãy un  giảm nên dãy tương ứng  
un 3  u 
n

2
cũng có giới hạn là .
3

37
Từ đó, ta tính được:

 2un1  5 n1 5    2  0  5 
un21
2
u
5un21  2un2un1  5unun 1
2 2
  
5
 lim 
 
2
un un 3 50
3un2  unun1 (4  un2 )
3
3  n1 (4  un2 ) 3  4  02
lim .
n  n  u 2 51
un 3

Đây chính là giới hạn cần tìm.

Nhận xét.

Trong bài toán trên, các biến đổi tương đối thuận lợi nhưng nhiều trường hợp khác, ta cần chú ý
đến định lí Taeplitz và Stolz liên quan đến tỉ số của hai dãy như sau:

* Định lí Taeplitz:

Cho Cn , k | 1  k  n; k , n     thỏa mãn các điều kiện:

i. Cn , k  0 khi n   .

n
ii. C
k 1
n,k  1 khi n   .

n
iii. C
k 1
n,k  C  const , n .

 n 
Khi đó, nếu dãy an  hội tụ thì dãy tương ứng bn   Cn , k ak  cũng hội tụ và hai dãy này có
 k 1

cùng giới hạn.

* Định lí Stolz:

Cho hai dãy số an  , bn  thỏa mãn:

i. bn tăng thực sự tới  .

an  an1
ii. lim c
n b  b
n n1

 a  a
Khi đó dãy  n  cũng có giới hạn và lim n  c .
 bn  n b
n

38
Bài 25. (Hà Tĩnh, vòng 2) Dãy số  xn  với n  1, 2, 3,... bị chặn trên và thỏa mãn điều kiện:

xn 2  xn 1  xn với mọi n  1, 2, 3,...


1 3
4 4

Chứng minh rằng dãy số trên có giới hạn.

Lời giải.

Đặt yn  min{xn , xn1}, n  1, 2,3,... . Ta sẽ chứng minh dãy này hội tụ. Thật vậy

1 3
xn 2  xn1  xn     yn  yn  yn  min{xn , xn1}  min{xn , xn 1 , xn  xn 1}
1 3 1 3
4 4 4 4 4 4
 min{xn1 , xn  2 }  yn 1

Theo giả thiết thì dãy  xn  bị chặn trên nên dãy  yn  cũng bị chặn trên. Đồng thời, theo nhận
xét trên thì dãy  yn  nên nó hội tụ. Đặt lim yn   .

 
Với mọi   0 , tồn tại N nguyên dương sao cho với mọi n  N thì:    yn    .
7 7


Theo định nghĩa của dãy  yn  , suy ra:    yn  xn .
7

  
- Nếu xn    thì suy ra:    yn    ,   0  lim xn   .

 
7 7 7
- Nếu xn    thì theo định nghĩa của  yn  , ta được xn1    (vì nếu ngược lại thì suy ra


7 7
rằng yn    ). Do đó, ta có
7
   
xn1  xn  xn1  xn  4 xn1  3 xn1  4      3         , tức là:
1 3
 7  7
    xn    ,   0 nên theo định nghĩa giới hạn thì dãy  xn  cũng hội tụ về  .
4 4

Vậy trong mọi trường hợp, dãy đã cho đều có giới hạn. Ta có đpcm.

Nhận xét.

Bài toán này đã từng xuất hiện trên tạp chí THTT. Một bài toán tương tự quen thuộc hơn đã có
trong đề chọn đội tuyển của Việt Nam 1988:

Dãy số  xn  với n  1, 2, 3,... bị chặn và thỏa mãn điều kiện: 2 xn  2  xn1  xn , n  1, 2,3,...
39
Chứng minh dãy này hội tụ.

*Các bài này đều có thể tổng quát lên thành:

Dãy số  xn  với n  1, 2, 3,... bị chặn trên và thỏa mãn điều kiện:

xn 2  pxn1  qxn , n  1, 2,3,... trong đó p  q  1, p, q  0

Chứng minh rằng dãy số trên có giới hạn.

(Nếu đổi điều kiện chặn trên thành chặn dưới thì điều kiện của dãy cũng tương ứng đổi thành
xn 2  pxn1  qxn ).

Bài 26. (Ninh Bình, vòng 2)


n
1
Chứng minh dãy un  xác định bởi công thức un    ln n có giới hạn hữu hạn.
k 1 k

Lời giải.

Trước hết, ta chứng minh bất đẳng thức ln  x  1  x, x  0 .

1 x
Thật vậy, xét hàm số f ( x )  ln  x  1  x, x  0 . Ta có f ( x )  1   0 nên đây là
x 1 x 1
hàm nghịch biến. Suy ra f ( x )  f (0)  ln1 0  0 .

Từ đây dễ dàng suy ra ln  x  1  x, x  0 .

Đẳng thức xảy ra khi và chỉ khi x  0 nên ln( x  1)  x, x  0 .

Trở lại bài toán, ta xét hiệu sau

1 1  n  1 1  1 
un1  un   ln n  1  ln n   ln     1    0
 n 
ln
n 1 n 1  n  n  1

Suy ra dãy đã cho đơn điệu giảm.

Mặt khác, ta cũng có


n
1 n
1  n
n 1
un    ln n   ln  1  ln n  ln(k  1)  ln k  ln n  ln 0
k 1 k k 1
 k  k 1 n

nên dãy này bị chặn dưới.


40
Dãy đã cho giảm và bị chặn dưới nên có giới hạn hữu hạn. Ta có đpcm.

Nhận xét.
n
1
Bài toán quen thuộc hơn thường được đề cập đến dãy số un   là chứng minh rằng nó tiến
k 1 k
n
1
tới vô cực và cách giải đơn giản nhất là dùng đánh giá un    ln n thông qua các biến đổi
k 1 k

logarit. Giới hạn của dãy trên có từ một kết quả nổi tiếng là hằng số Euler C.
2n
1
* Một bài tương tự: Tìm giới hạn của dãy số sau un  
k n 1 k
, n  1, 2, 3,...

Bài 27. (Hà Nội, vòng 2)

Cho dãy số nguyên dương U n  thoả mãn U1  1, U 2  2, U 4  5 và với mọi n  1 thì

U n1U n1  U n2  a với a 2  1 .

1) Xác định số hạng tổng quát của dãy số trên.


2) Tìm các số tự nhiên n không vượt quá 2012 sao cho U n chia hết cho 10.

Lời giải.

1) Từ công thức xác định dãy, ta có U 32  U 2U 4  a  10  a , do U 3 nguyên dương nên a  1 và


U3  3 .

U n2  a
Bằng quy nạp, ta thấy rằng dãy này tăng và U n  3, n  3 . Ta có U n1  .
U n1

Giả sử rằng U n2  1U n1 và U n2 1U n1 thì 2U n1 , vô lí vì đây là dãy số nguyên dương và theo
nhận xét trên thì U n  3, n  3 .

Do đó, có không quá một dãy số thỏa mãn đề bài.

 F1  1, F2  2
Ta xét dãy Fibonacci xác định như sau 
 Fn 2  Fn1  Fn , n  1, 2,3,...

Bằng quy nạp, ta cũng có thể chứng minh được rằng Fn21  Fn Fn 2  (1) n , n .

41
Dãy số này cũng thỏa mãn đề bài nên hai dãy U n  và  Fn  là trùng nhau.

Từ đó, ta tìm được công thức tổng quát của dãy U n  dựa theo dãy Fibonacci là

 n
1 1  5  1 5  
n

Un        .
5  2   2  
 

2) Ta có U n 2  U n1  U n  2U n  U n1  U n1 (mod 2) và

U n 2  2U n  U n1  3U n1  2U n2  5U n2  3U n3  3U n3 (mod 5) (do (3,5)  1) .

Theo công thức xác định dãy thì U 2  2, 0  U 2  U 5  U 8  ...(mod 2) nên bằng quy nạp, ta có
thể chứng minh được rằng U n  0(mod 2)  n  3k  2, k   .

Tương tự, do U 4  5 nên cũng bằng quy nạp thì U n  0(mod 5)  n  5k  4, k   .

n  2(mod 3)
Từ đó suy ra U n  0(mod10)    n  14(mod15)  k  * , n  15k 1 .
n  4(mod 5)

Do 1  n  2012 nên để tìm tất cả các giá trị n thỏa mãn đề bài, ta xét

1  15k 1  2012  1  k  134 .

Vậy có đúng 134 số hạng thỏa mãn đề bài.

Nhận xét.

Vấn đề ở bài này chính là việc phát biểu a trong đề bài dễ gây nhầm lẫn rằng a là hằng số và
nhiều bạn sẽ từ bước tìm điều kiện U 3 nguyên dương để suy ra a  1 thì công nhận rằng giá trị
đó không đổi với những quan hệ khác trong công thức tổng quát dẫn đến sai lầm.

Tuy nhiên, ý của đề bài lại chính là a có thể nhận giá trị là 1 hoặc 1 trong từng quan hệ, miễn
sao giá trị U n1 nguyên dương là được.

Nói về câu 2 của bài toán này, ta cũng có nhiều kết quả thú vị. Chẳng hạn như F32n1  2n , n hoặc
F5n  5n trong trường hợp dãy xác định theo kiểu F1  F2  1, Fn 2  Fn1  Fn , n  1 .

Trong cuốn tuyển tập 30 năm Tạp chí THTT, có một bài báo cũng nhắc đến đến bài toán này và
câu chuyện về một học sinh trong kì thi Vô địch Toán của Liên Xô năm 1946 (tương đương với
kì thi quốc gia) đã phát biểu và chứng minh bài toán:
42
Tìm số hạng đầu tiên trong 104 số hạng đầu của dãy Fibonacci mà tận cùng bằng 4 chữ số 0.

Trong đề thi có nhiều bài toán nhưng bằng việ chứng minh một kết quả tổng quát tuyệt vời cho
bài toán này mà cậu học sinh đã giành giải nhất trong kì thi đó. Bài toán năm đó cậu đã tổng quát
lên từ bài toán gốc của đề và giải được là

Hãy tìm tất cả các số hạng của dãy Fibonacci có tận cùng bằng 4 chữ số 0.

Muốn chứng minh kết quả này, cần phải chứng minh được rằng chỉ có những số hạng có dạng
F3 p242  F12 p mới chia hết cho 24 và chỉ những số hạng có dạng F54 p  F625 p mới chia hết cho
54 , từ đó suy ra tất cả số hạng cần tìm có dạng F7500 p , p   .

Để giải bài toán này, đòi hỏi cần phải chịu khó tìm tòi, khám phá và kiên nhẫn!

Bài 28. (KHTN, vòng 3)

Cho dãy số dương an  thỏa mãn


2 1 3
a1  1, a2  , an2  an21  an , n  1, 2, 3,...
3 4 4

Chứng minh rằng an hội tụ và tìm giới hạn của nó.

Lời giải.

2
Ta có a1  1, a2  nên bằng quy nạp, ta có thể chứng minh được rằng 0  an  1, n .
3

1 3 1 3
Từ đó suy ra an 2  an21  an  an1  an , n .
4 4 4 4

Dãy số này bị chặn và thỏa mãn hệ thức trên nên tương tự bài 23, ta có thể chứng minh nó có
giới hạn.

2 1 3
Xét dãy số tương ứng là b1  1, b2  , bn 2  bn21  bn , n  1, 2, 3,...
3 4 4

Dễ dàng thấy rằng an  bn , n  2 . Ta chứng minh được rằng lim bn  0 .

Từ đó, ta thấy rằng 0  an  bn , n  2 và lim bn  0 nên theo nguyên lí kẹp thì lim an  0 .

Ta có đpcm.

Nhận xét.
43
Các bài dãy số có công thức truy hồi cấp 2, 3 thế này thuộc dạng khó và để đánh giá nó, đòi hỏi
khá nhiều kĩ thuật. Bài toán này không những thế, còn kết hợp thêm dạng dãy số xác định không
theo đẳng thức và khiến cho vấn đề trở nên rắc rối hơn nhiều.
Dạng này cũng đã từng xuất hiện trong đề chọn đội tuyển Việt Nam năm 1990 và 1991:
1) Đề TST 1990:
Cho bốn số thực dương A, B, a, b . Xét dãy số thực ( xn ) thỏa mãn:
 x1  a, x2  b

 xn 1  A 3 xn  B 3 xn 1 , n  2
2 2

Chứng minh rằng tồn tại giới hạn lim xn và tìm giới hạn đó.
n 

Cho dãy thực dương  xn  được xác định bởi


2) Đề TST 1991:

x1  1, x2  9, x3  9, x4  1, xn 4  4 xn .xn1.xn 2 .xn 3 , n  1 .


Chứng minh dãy này có giới hạn hữu hạn. Tìm giới hạn đó.

Bài 29. (Chọn đội tuyển ĐHSP Hà Nội)

2n  3
Cho dãy số an  , n 1 thỏa mãn: a1  1, an  an1 , n  2 và dãy bn  thỏa mãn
2n
n
bn   ai , n  1 . Chứng minh dãy bn  có giới hạn hữu hạn, tìm giới hạn đó.
i 1

Lời giải.

Từ công thức xác định dãy đã cho, ta có 2nan  2n  3 an1  an1  2 (n 1)an1  nan , n  1 .

Do đó, ta có
n n
bn   ai   2 iai  (i  1)ai1   2 1  (n  1)an1  .
i 1 i 1

1
Ta sẽ chứng minh bằng quy nạp rằng nan  , n . Thật vậy,
n

- Với n  1 , ta có a1  1 nên khẳng định đúng.

1
- Giả sử khẳng định đúng với n  k  1 , tức là kak  .
k
44
2(k  1)  3 2k 1 1
Ta có ak 1  ak  , ta cần chứng minh
2(k 1) 2k  2 k k
2 k 1 1 1
  2k 1 k  1  2k k  (4k 2  4k  1)(k 1)  4k 3  1  3k .
2k  2 k k  k 1 k  1

Bất đẳng thức cuối đúng nên khẳng định trên cũng đúng với n  k  1 .

Theo nguyên lí quy nạp thì khẳng định được chứng minh.

 1 
Từ đó suy ra 2 1   2 1  (n  1)an1   bn  2 .
 n  1 

Theo nguyên lí kẹp thì dãy bn có giới hạn và giới hạn đó là 2.

Nhận xét.

2n  3 (2n  3)(2n 1)...5  3


Bài này có thể từ công thức truy hồi an  an1  an  .
2n 2n(2n  2)...4  2

 (2n  3)(2n  5)...5  3 


2

Suy ra a  
2
  22n  3 2 (2n  3)(2n 2 5) ... 5 23 3 21  22n  3 2 .
n
 2n(2n  2)...4  2  4n (2n  2) (2n  4) 4 2 4n (2n  2)

2n  3
Từ đó ta cũng có một bất đẳng thức tương tự lời giải ở trên 2nan  .
2n  2

Ngoài ra, việc đánh giá dãy nan có thể dùng định lí Stolz.

Bài 30. (Đại học KHTN Hà Nội, vòng 1)

Cho dãy số an  xác định như sau


a1  6, a2  14

an 2  6an 1  an  24.(1) , n  1, 2,3,...
n

a
n
1
Tính giới hạn lim .
n 
k 1 k

Lời giải.

Xét phương trình x 2  6 x  1  0 có hai nghiệm là

 
x1  3  2 2  1  2 , x2  3  2 2  1  2 .  
2 2

45
Phương trình sai phân ở trên có nghiệm tổng quát là:

an  1 3  2 2  
 2 3  2 2    3  1 .
n n n

Thay vào công thức truy hồi, ta được:  3  1  6 3  1   3  1  24  1 hay
n2 n 1 n n

 1 83  24   0 .
n

Công thức này phải đúng với mọi n nên  3  3 .

Tiếp tục so sánh với điều kiện a1  6, a2  14 , ta tính được hai giá trị của tham số tương ứng là

1   2 
1
. Do đó, công thức tổng quát của dãy là:
2

an 
1
2 
3 2 2   3  2 2    3  1 , n .
n n n


Đặt bn  1  2   1  2  , n  1, 2,3,...
n n
Ta có:

bnbn 2   1  2   1  2   1  2  


 1 2  
n 2 n2

 
n n


 3 2 2   3  2 2   1  2  1  2  1  2   1  2   .
n n n n 2 2

  3  2 2    3  2 2   6(1)
n n
n

Từ biến đổi trên, ta cũng có


bn21  bn1bn 3   1  2    
   1  2 n 1  1  2 n1   1  2 n 3  1  2
 1 2        
n 1 n 1 2 n 3

     


 2 1 2   1 2  
  1 2  1 2  1 2  
1 2  
 32( 1) n 1   
n 1 n 1 n 1 n 1

 
3 3

Từ đó suy ra

16(1) n 32( 1) n bn21  bn1bn3 bn 1 bn 3


    , n  2, 3, 4,...
an bn1bn 1 bn1bn 1 bn1 bn1

Ta xét hai trường hợp sau:


-Nếu n chẵn, đặt n  2k , k    . Ta có:
46
    2 k 1  2 k 3   3  2 n 3 .
b  b b
 
16 b2 k 1 b2k  3 n
16 n
b
a2 k b2 k 1 b2k 1 k 1 a2 k k 1  b2 k 1 b2 k 1  b1 b2 n 1

-Nếu n lẻ, đặt n  2k  1, k    . Ta có:

 2k  4  2 k  2      2 k  4  2k  2
b  b2 n 4 b2
  .
n n
16 b b 16 b
a2 k 1 b2k  2 b2 k k 1 a2 k 1 k 1  b2 k  2 b2 k  b2 n  2 b0

Dễ dàng thấy rằng lim  2n 3  2 n 4   0 và 3  2  4 nên lim   4  lim   .


b b  b b n
16 n
1 1
 2 n1 b2 n 2 
n  b b1 b0 n 
i 1 ai
n 
i 1 ai 4
1
Vậy giới hạn của dãy đã cho là .
4
Nhận xét.

các năm gần đây là từ một dãy số xn suy ra giới hạn của một dãy  yn  khác được xác định bằng
Bài toán này cũng có hình thức tương tự như nhiều bài toán về giới hạn dãy số xuất hiện trong

tổng yn   f  xi  với f ( x ) là một hàm số nào đấy. Cách giải chung cho dạng này vẫn là từ
n

i 1

công thức của  xn  , biến đổi và đưa về sai phân để rút gọn tổng trên; từ đó xác định được biểu
thức đơn giản hơn của yn .

Tuy nhiên, ở bài toán này, ta thấy rằng việc biến đổi đó không còn dễ dàng được nhận ra nữa.
Bằng cách xét thêm dãy số phụ và dùng nó để xử lí được lũy thừa (1) n , ta đã rút gọn được tổng
đã cho ở dạng đặc biệt và xác định được giới hạn.

Chú ý thêm rằng tổng có dạng như trên còn gọi là chuỗi số và chuỗi có giới hạn hữu hạn gọi là
chuỗi hội tụ (trường hợp ngược lại là chuỗi phân kì).

47
C – MỘT SỐ VẤN ĐỀ VỀ BIỆN LUẬN GIỚI HẠN DÃY SỐ

Các bài toán về biện luận dãy số là một dạng khó và xuất hiện khá nhiều trong các đề thi HSG,
việc tham số hóa trong việc xác định các số hạng của dãy làm cho bài toán dãy số thông thường
trở nên thú hơn khá nhiều. Để có thể giải quyết được các bài toán này, chúng ta cần phải có một
số kiến thức căn bản để có thể tiếp tục có những ý tưởng rõ ràng hơn cho việc biến đổi và phân
tích vấn đề. Chúng ta sẽ cùng xem xét một số bài toán liên quan đến các dạng này cùng một số
phương pháp cơ bản để giải quyết chúng.

Trong phần này, chúng ta công nhận các kết quả quen thuộc về giới hạn của dãy như: dãy đơn
điệu và bị chặn thì hội tụ, dãy các đoạn thắt lồng nhau có đúng một giao điểm chung thì hội tụ,…

 x  a
Các bài toán về khảo sát giới hạn của dãy  1 , n  1, 2,3,... thông thường được thực
 xn1  f ( xn )
hiện dựa theo tính chất và sự biến thiên của hàm số f ( x ) . Trên miền xác định của nó, số nghiệm
của phương trình f ( x )  x có ảnh hưởng trực tiếp đến sự thay đổi của hàm số và tính chất có
giới hạn của dãy.

Ta biết rằng nếu f ( x ) đồng biến trên miền xác định của nó hoặc đồng biến trên miền giá trị của
các số hạng của dãy thì dãy số đã cho sẽ đơn điệu. Ta có các trường hợp sau

- Nếu x1  x2 thì f ( x1 )  f ( x2 )  x2  x3 và tương tự, ta chứng minh được rằng

x1  x2  x3  ...  xk  xk 1  ...

tức là dãy này đồng biến.

- Nếu x2  x1 thì tương tự, ta có dãy nghịch biến.

- Nếu x1  x2 thì dãy này không đổi.

Do đó, sự biến thiên của dãy số trong trường hợp này phụ thuộc hoàn toàn vào các khoảng
nghiệm của phương trình f ( x )  x .

Và để khảo sát được tính hội tụ của dãy thì ta chỉ cần giải được phương trình này và tìm các chặn
trên, chặn dưới tương ứng là xong. Dãy số trong trường hợp này dễ xử lí.

Trường hợp f ( x ) là hàm đơn điệu giảm, dễ dàng thấy rằng f  f ( x ) là hàm đơn điệu tăng. Khi
đó, dựa theo kết quả phần trên thì từ việc so sánh f  f ( x ) với x, ta sẽ xác định được tính tăng
giảm của hai dãy con  x2n  và  x2 n 1  .

48
Hai dãy này đơn điệu ngược chiều nhau vì x2 n  f  x2 n1  , x2 n1  f  x2 n  ,... Trong một số
trường hợp, hàm số đã cho không đơn điệu trên cả tập xác định mà chỉ đơn điệu trên miền giá trị
mà các số hạng của dãy nhận được. Ta cần xác định miền đó càng hẹp càng tốt để trên đó, hàm
số đã cho đơn điệu và áp dụng phương pháp đánh giá này trên dãy số đã cho.

Các bước cụ thể của cách giải dạng này được giới thiệu trong ví dụ sau.

Ví dụ 1.

u1   0;1

Cho dãy số (un ) thỏa mãn:  un2  4un  1
un 1  u 2  u  1 , n  1
 n n

Chứng minh dãy số (un ) có giới hạn hữu hạn. Tìm giới hạn đó.

Lời giải.

un2  4un  1 (un  1)2


Ta thấy un  0, n và từ: un 1      , ta có: 1  un  2, n .
3un
un2  un  1 un2  un  1 un2  un  1
1 2

x2  4x  1 3(1  x 2 )
Xét hàm số: f ( x)       0 .
x2  x  1 x2  x  1
, x 1; 2 f '( x )

Suy ra là hàm này nghịch biến trên 1; 2  .

u1  0;1
Dãy số đó cho có thể viết dưới dạng: 
un1  f un  , n  1, 2, 3,...

Ta thấy: u1  u3  f u1   f u3   u2  u4  f u2   f u4   u3  u5

Tiến hành tương tự, suy ra:

u1  u3  u5  ....  Dãy u2 n 1 tăng và bị chặn trên bởi 2 nên có giới hạn, giả sử là   1; 2 .

u2  u4  u6  ...  Dãy u2n giảm và bị chặn dưới bởi 1 nên có giới hạn, giả sử là   1; 2 .

u  f (u 2 n )   f (  )
Ta có:  2 n 1 
 2n  2     
Chuyển qua giới hạn, ta có .
u f (u 2 n 1 ) f ( )

49
 2  4  1  2  4  1
     f ( )  f ( )      2 
   1  2   1
    3(   )(  1)
     3 2  2     2
    1    1  (    1)(  2    1)
    0

3(  1)  (    1)(     1)
Ta thấy phương trình thứ hai không có giá trị  ,   1; 2 thỏa mãn     t .
2 2

Do đó, lim u2 n 1  lim u2 n  t , hai dãy con đó có cùng giới hạn là t.


n  n 

t 2  4t  1
Ta thấy, t phải thỏa mãn đẳng thức: t  2  t 3  3t  1
t  t 1
(*).

Ta sẽ chứng minh rằng nghiệm t  2 . Đặt t  2 cos  ,    0; 2  , thay vào phương trình (*):

 2
8cos3   6 cos   1  cos 3     k . Do    0; 2  nên
1
2 9 3
 5 7  5 7
 ; ; , tương ứng với các nghiệm của (*) là: t  2 cos ;2cos ; 2cos .
9 9 9 9 9 9
Phương trình (*) đó có đủ 3 nghiệm nên nó không có nghiệm t  2 .

Trong các nghiệm này, chỉ có t  2 cos  1; 2 thỏa mãn và đây cũng chính là giới hạn cần tìm.


9
Vậy dãy số un có giới hạn hữu hạn và lim un  2cos .
n  9
Nhận xét. Như vậy các dãy số có dạng xn 1  f ( xn ) và hàm f ( x ) đơn điệu gần như đã được
giải quyết hoàn toàn. Tuy nhiên, trong các trường hợp khác, dãy này không đơn điệu thì việc
biện luận phức tạp hơn nhiều. Ta tiến hành khảo sát dãy đã cho và dùng nguyên lí kẹp để suy ra
giới hạn. Các bước giải bài toán này được thể hiện trong ví dụ sau:

Ví dụ 2.

un  k
Cho dãy số xác định bởi công thức u1  a  (0,1), un 1  , k  1, n  1, 2, 3,...
k .un  1
9

Chứng minh rằng lim un  1 .

Lời giải.

Từ công thức xác định của dãy số, bằng quy nạp, ta có được un  (0,1), n .
50
xk
Xét hàm số f ( x)  9 , k  1, x  [0,1] . Ta có
kx  1
 kx  1
8
1 1 k 2  1 k 2 1
f ( x )  9    f ( x)  , x  0;1
9 (kx 1)2  x  k  9(kx 1) 9 (kx  1)( x  k )8 9
Nếu dãy đã cho hội tụ thì giới hạn của nó phải là nghiệm của phương trình
xk
f ( x)  x  x  9  kx10  x9  x  k  k ( x10  1)  x( x8  1)  0  x  1  0 .
kx  1
f ( x)  f (1) 1
Theo định lí Lagrange thì tồn tại c thuộc (0,1) :  f  (c)  f ( x)  f (1)  x 1
x 1 9

Ta cũng có f (1)  1 nên un1  1  f (un )  1  un  1  ...  n a  1 .


1 1
9 9
Theo nguyên lí kẹp thì lim(un  1)  0 hay lim un  1 .

Nhận xét. Trường hợp còn lại cần quan tâm là dãy số xác định theo công thức tương tự trên
nhưng hàm số f  x  không đơn điệu và không thỏa mãn tính chất 0  f ( x)  q  1 . Khi đó, ta
phải dựa vào sự tương giao giữa đồ thị hàm số y  f ( x) và hàm số y  x để biện luận giới hạn
dãy, khảo sát tính hội tụ của dãy. Ta xét hàm số cho bởi đồ thị minh họa dưới đây:

Giả sử ta biểu diễn các số hạng của dãy trên trục hoành của hệ trục tọa độ . Ta thấy rằng đường
thẳng y  x cắt đồ thị (C) tương ứng tại ba điểm phân biệt, giả sử chúng có hoành độ lần lượt là
x1  x2  x3 . Khi đó, các số hạng của dãy số sẽ được xác định bằng cách từ một điểm bất kì
thuộc trục hoành, đặt là u1 . Ta vẽ đường thẳng song song với trục tung cắt đồ thị (C) tại một

51
điểm, tung độ của điểm đó chính là u2  f (u1 ) . Ta lại vẽ đường thẳng qua điểm này và song
song với trục hoành, cắt đường thẳng y  x tại một điểm, hình chiếu của điểm đó lên trục hoành
chính là vị trí của u2 . Cứ tiếp tục như thế, ta xác định được vị trí của tất cả các số hạng của dãy.
Khi đó, dãy đã cho hội tụ khi các vị trị này tiến về một điểm nào đó trên trục hoành. Trong hình
vẽ dưới đây, các đường màu xanh ở bìa phải cho ta minh họa về một dãy số có u1 lớn hơn
nghiệm lớn nhất của phương trình là x3 , đường màu hồng ở bìa trái cho ta minh họa về một dãy
số có u1 bé hơn nghiệm nhỏ nhất của phương trình là x1 . Dễ thấy rằng hai đường này không cho
ta một dãy hội tụ nào cả. Trong trường hợp số hạng u1 trùng với một trong ba nghiệm đã nêu thì
ta có dãy hằng nên tất nhiên nó hội tụ. Do đó, thông thường thì một dãy hội tụ khi u1  ( x1 , x3 ) và
giới hạn tương ứng chính là x2 (vẫn có trường hợp thuộc miền đó nhưng không hội tụ).

Đây là xét riêng trong trường hợp phương trình f ( x )  x có ba nghiệm phân biệt. Các trường
hợp khác thì tùy vào số nghiệm và chiều biến thiên của hàm số trên từng khoảng mà ta có các kết
quả khác nhau tương ứng.

Ta sẽ xem xét lời giải bài này qua ví dụ sau

52
Ví dụ 3.
Tìm điều kiện của a sao cho dãy số xác định như sau hội tụ:
x1  a, xn1  xn2  2 xn  2, n  1, 2,3,...
Lời giải.
Xét hàm số tương ứng là f ( x )  x 2  2 x  2 với đồ thị minh họa trong hình bên dưới.
Phương trình f ( x )  x  x 2  3x  2  0 có hai nghiệm phân biệt là x1  1, x2  2 nên
f ( x )  x với mọi x  1  x  2 và f ( x )  x với 1  x  2 .

Ta xét các trường hợp sau:


-Nếu a  1 thì xn  1, n nên dãy đã cho hội tụ.

-Nếu a  2 thì xn  2, n nên dãy đã cho cũng hội tụ.

-Nếu a  2 thì x2  f (a)  a  x1 và đồng thời hàm số f ( x )  2 x  2  0 nên đồng biến. Khi
đó, dãy này tăng và không bị chặn trên nên không hội tụ (vì nếu dãy hội tụ thì nó phải hội tụ về
hai nghiệm của phương trình f ( x )  x nhưng điều này không thể xảy ra).

-Ta thấy rằng chỉ còn cần xét a  1; 2 vì rõ ràng nếu a  1 thì x2  f ( x1 )  a 2  2a  2  1 và
ta có thể tiến hành khảo sát dãy từ vị trị x2 . Ta thấy khi a  1; 2 thì

x2  f (a)  a 2  2a  2  a(a  2)  2  2

Bằng quy nạp, ta chứng minh được rằng xn  1; 2 , n .

53
Khi đó, hàm số đã cho, xét trên miền (1;2) có đạo hàm f ( x)  2 x  2  0 nên đồng biến, đồng
thời x2  f ( x1 )  x1 nên dãy này giảm và bị chặn dưới bởi 1 nên hội tụ. Gọi giới hạn đó là l thì
ta có l  [1; 2) và l  l 2  2l  2 hay l  1 .
-Ta sẽ tìm điều kiện để tồn tại một giá trị xk của dãy sao cho xk  1; 2 , khi đó

 x 2  2 xk  1  0
1  xk 1  2  1  xk2  2 xk  2  2   k2  0  xk  2 .
 xk  2 xk  0

Điều kiện ở trên lại có thể viết thành 0  xk21  2 xk 1  2  2  0  xk 1  2 .
Do đó, nếu như tồn tại một giá trị của dãy sao cho xk  [0; 2] thì tất cả số hạng kể từ đó cũng sẽ
thuộc miền này, nói riêng , ta có điều kiện a  [0; 2] .
Vậy các giá trị cần tìm của a là a  [0; 2] .
Nhận xét.
Một số trường hợp khác hàm số xác định dãy có dạng phân thức và không những ta cần phải
biện luận giới hạn của dãy mà còn phải tìm điều kiện xác định của tất cả các phần tử của dãy.
Khi đó, ta cần phải vận dụng tính đơn điệu và song ánh của hàm số để dùng hàm ngược và xây
dựng các dãy tương ứng mà nếu các phần tử của dãy ban đầu nhận một trong các giá trị đó thì
đến một lúc nào đó sẽ không xác định, dãy đã cho chỉ cần tránh đi các phần tử đó thì tất cả các số
hạng đều xác định. Việc biện luận giới hạn dãy trong trường hợp này cần phải cẩn thận, tập trung
khai thác các tính chất đặc trưng của hàm số để đơn giản hóa các bước tính toán và phân tích vấn
đề. Dưới đây chúng ta sẽ phân tích lời giải một bài toán minh họa cho trường hợp này.
Ví dụ 4.
2 xn3
Cho dãy số  xn  thỏa mãn x1  a, xn1  2 , n  1, 2,3,...
3xn 1

Hãy tìm tất cả các giá trị a sao cho dãy này xác định và có giới hạn hữu hạn.

Lời giải.

2 x3 3
Đặt f ( x)  ,x . Ta có u1  a, un1  f (un ), n  1, 2,3,...
3 x 1
2
3

6 x 4  6 x 2 6 x 2 ( x 2 1)
Ta có f ( x )   .
(3x 2 1)2 (3x 2 1)2
 3  3 3   3 
Suy ra hàm này tăng trên (;1]  [1; ) và giảm trên 1;     ;    ;1 .
 3   3 3   3 

54
Do f ( x ) là hàm số lẻ nên nếu thay x1  a thì tất cả số hạng của dãy sẽ đổi dấu. Vì thế, ta chỉ
cần xét đặc trưng trên miền dương của tập xác định.

Nếu x1  a  0 thì tất cả các số hạng của dãy là 0 nên thỏa mãn yêu cầu đề bài. Ta xét a  0 ,
dựa vào sự biến thiên của hàm f ( x ) , ta sẽ tìm điều kiện của giá trị a .

Vì f (0)  0, lim    nên do tính đơn điệu và liên tục của hàm f ( x ) thì với c  (;0]
 3 
x 
 3 

 3
thì tồn tại duy nhất x  0;  sao cho f ( x )  c .
 3 

 3
thì tồn tại duy nhất a1   0;  sao cho f  a1   a0 , tương tự tồn tại duy nhất
3
Ta đặt a0 
3  3 
 3
a2   0;  sao cho f  a2   a1 . Tương tự, ta xây dựng các số hạng cho dãy an  vừa nêu.
 3 

 3
Ta thấy a1  a0 và hàm số f ( x ) đơn điệu giảm trên miền  0;  nên
 3 

f (a0 )  f (a1 )  a1  a2  a1  a2

Tương tự với các số hạng còn lại, tức là dãy này đơn điệu giảm. Hơn nữa, dãy này bị chặn dưới
2 x3
bởi 0 nên có giới hạn, đặt là l . Ta thấy f ( x)  x  0  2 x0 x 5 .
3 x 1

3 5 3
Xét f ( x )  x trên miền 0  x  thì f ( x )  x  0 khi x và f ( x )  x  0 khi
3 5 3
 5 3
. Do đó, f (an )  an  an  an1  0 nên an  ;  , suy ra lim an 
5 5
0 x .
5  5 3  5

Từ việc khảo sát dãy số này, ta có được điều kiện sau:

- Nếu tồn tại n sao cho a  an thì có hai trường hợp, nếu a  an thì
x2  f ( x1 )  f (an )  an1 ; còn nếu a  an thì x2  f ( x1 )  f (an )   f (an )  an1 .

3
Tức là nếu a  an thì x2  an1 , x3  an2 ,... hay xn1  a0  và  xn  không xác định.
3

55
5
- Còn nếu a  thì dễ thấy rằng x2  x1 , x3  x2 , x4  x3 ,... và dãy này không có
5
giới hạn.

5
Ta sẽ chứng minh rằng nếu a  an , n và a  thì dãy đã cho xác định và hội tụ.
5

Thật vậy, ta có các trường hợp sau:

- Nếu a  0 thì tất cả các phần tử của dãy  xn  đều bằng 0 nên hội tụ.

5
- Nếu 0  a  thì 0  f (a )  a nên x2  x1 . Suy ra dãy  xn  giảm và bị chặn dưới
5
5
bởi 0 nên có giới hạn, đặt là c. Do 0  a  và f (c )  c nên c  0 . Do đó, trong
5
trường hợp này thì dãy có giới hạn là 0.

- Nếu tồn tại k sao cho xk  1 thì tất cả các số hạng của dãy từ đó trở đi đều bằng 1 nên
3
cũng hội tụ. Còn nếu tồn tại k sao cho  xk  1 hoặc xk  1 thì do f ( x ) giảm trên
3
 3 
 ;1 và tăng trên [1; ) nên x  1 . Dễ thấy 1  f ( x)  x, x  1 nên từ số hạng
 3  k 1
 
này trở đi thì dãy giảm và bị chặn dưới bởi 1 nên hội tụ. Từ các điều này, ta suy ra nếu
3 3
a thì dãy đã cho hội tụ. Do đó, dãy đã cho cũng hội tụ với a  .
3 3

5 3
- Nếu a và a  an , n thì f (a)  ak 2 , ak 1   x2  f ( x1 )   ak 1 , ak 2  .
5 3
3
Tương tự, x3   ak 2 , ak 3  và suy ra được rằng xk   a1 , a0  và cuối cùng ak 1  .
3
Trường hợp này đã được xét ở trên nên dãy đã cho cũng hội tụ.

3 3
Nói tóm lại, dãy  xn  xác định và có giới hạn hữu hạn khi và chỉ khi a  hoặc a  và
3 3
a  an , n với dãy an  xác định như trên.

56
D – MỘT SỐ BÀI TOÁN CHỌN LỌC VỀ GIỚI HẠN DÃY SỐ

Bài 1.

Cho dãy số an  xác định bởi a0   , an  an21  an1 , n  1 . Hãy tìm tất cả các giá trị của a0 để
dãy số này hội tụ.

Lời giải.

Ta thấy rằng nếu tồn tại số k nào đó mà 0  ak  1 thì

1 1
  ak 1  0, 0  ak 2  1,   ak 3  0, 0  ak 4  1,...
4 4

Khi đó, dãy ak  2l  sẽ đơn điệu giảm vì

ak  2l 2  ak 2 l  ak22 l 1  ak  2l 1  ak 2 l  ak42 l  2ak3 2l  ak3 2l (ak  2l  2)  0

Dãy này cũng bị chặn dưới nên tồn tại giới hạn c  lim ak 2 l . Chuyển đẳng thức tương ứng ở
l 

trên qua giới hạn thì ta cần xét phương trình c  c  c 4  c 3 với c  1 thì ta được c  0 .

Dãy ak 2 l 1  tương tự cũng sẽ đơn điệu giảm và bị chặn dưới nên có giới hạn. Ta có

lim ak  2 l 1  lim  ak2 2l  ak 2 l   c 2  c  0


l  l 

Suy ra lim an  0 .

Tiếp theo, ta sẽ chứng tỏ rằng tồn tại giá trị k sao cho 0  ak  2 , khi đó ak 1  ak  2 . Nếu
ak 1  0 thì ta lại có ak  2  ak 1  2 và tương tự như vậy. Do đó, hoặc là dãy này gồm các phần
tử không âm và đơn điệu tăng, tức là nó có giới hạn hoặc là với l  1 nào đó mà ak 1  0  ak l 1 .
Như phần chứng minh ở trên, ta thấy rằng khi đó 0  ak l 1  1 và dãy này cũng có hội tụ.

Do đó, ta đã chứng minh được rằng dãy đã cho hội tụ khi 0  a1  2 .

Ta xét bất phương trình 0  a2  2  0  a12  a1  2  1  a1  0  1  a1  2 .

Do đó, ta có thêm điều kiện là 1  a1  0 thì dãy sẽ có giới hạn hữu hạn.

Ta sẽ chứng minh rằng các trường hợp còn lại sẽ dẫn đến một dãy không có giới hạn hữu hạn.

57
Thật vậy, xét a1  1  a1  2 thì a2  a12  a1  2 và a3  a2  a2 (a2  2)  a2 .

Tương tự với các giá trị tiếp theo, tức là ta đã chứng minh được rằng dãy này tăng thực sự.

Nếu tồn tại c sao cho lim an  c thì c 2  c  c  c  0  c  2 , nhưng những giá trị này không
thể là giới hạn của dãy đã cho được vì an  a2  2, n và dãy đã cho không có giới hạn trong
trường hợp này.

Vậy dãy đã cho hội tụ khi và chỉ khi 1  a1  2 .

Bài 2.

Khảo sát sự hội tụ của dãy số un  xác định bởi u0  a  0, un 1 


2
1  un2
.

Lời giải.

Bằng quy nạp, ta có thể chứng minh được rằng un  0 với mọi n.

Xét hàm số f :[0; )  [0; ) , f ( x) 


2
1  x2
là hàm liên tục. Ta có:

x  [0; ), f ( x )  x  x 3  x  2  0  ( x  1)( x 2  x  2)  0  x  1

Do đó, nếu un  hội tụ thì nó chỉ có thể hội tụ đến 1.

Hàm số f khả vi trên [0; +) và x  [0; ), f '( x)    0 , suy ra f giảm.
4x
(1  x 2 ) 2

Tiếp theo, ta sẽ chứng minh lim u2 k  1 và lim u2 k 1  1 .

2(1  x 2 ) 2
Xét g  f  f :[0; )  [0; ) , g ( x) 
(1  x 2 ) 2  4
thì g là một hàm tăng vì f giảm.

x5  2x 4  2 x3  4x 2  5x  2 ( x  1) 3 ( x 2  x  2)
Ta tính g ( x )  x    
(1  x 2 ) 2  4 (1  x 2 ) 2  4

Ta xét các trường hợp sau:

*Trường hợp 1: u0  a  [0,1] . Khi ấy với mọi k   , ( u2 k  [0,1] và u2 k 1  [1; ] ) Vậy với mọi
k   , ta có

58
u 2 k  2  u2 k  g ( u 2 k )  u 2 k  0

u2 k 3  u2 k 1  g (u2 k 1 )  u2 k 1  0

Do đó u2k  tăng và u2 k 1 giảm.

Hơn nữa, vì ( p   , u2 k  1  u2 k 1 ), nên ta suy ra rằng u2k  hội tụ đến một giới hạn L1 thuộc
[0; ) và u2 k 1 hội tụ đến một giới hạn L2 thuộc [0; ) .

Vì g liên tục trên [0; ) và vì phương trình g ( x)  x có nghiệm duy nhất x  1 trên [0; ) nên ta
suy ra L1  L2  1 . Cuối cùng ta được lim un  1 .

*Trường hợp 2: u0  a  [1,  ] :

Vì u1  f (u0 )  f (a)  [0,1] ta quy về trường hợp trên (bằng cách thay u0 bởi u1) và có cùng một
kết luận lim un  1 .

Bài 3.

Tìm tất cả các giá trị của a để dãy số  xn  xác định bởi x0  a, xn 1  2  xn2 có giới hạn hữu hạn.

Lời giải.

Hàm số f ( x )  2  x 2 tăng trên (, 0) và giảm trên (0, ) .

Phương trình f ( x )  x có hai nghiệm là x  2 và x  1 . Ta xét các trường hợp sau:

- Đầu tiên, nhận xét rằng nếu a  2 thì do f : (, 2)  (, 2) và là hàm tăng, hơn nữa
x1  2  a 2  x0 nên dãy số  xn  giảm. Nếu dãy  xn  bị chặn dưới thì nó hội tụ về nghiệm của
phương trình f ( x )  x , điều này mâu thuẫn vì dãy giảm và x0  2 .

Do đó, trong trường hợp này  xn  không bị chặn dưới, tức không có giới hạn hữu hạn.

Nếu a  2 thì x1  2 và ta cũng suy  xn  không có giới hạn hữu hạn.

- Với a  2 hoặc a  1 thì dãy số có giới hạn vì đều là các dãy hằng. Xét x0   2, 2 .

Ta chứng minh dãy số có giới hạn hữu hạn khi và chỉ khi tồn tại một giá trị n nào đó sao cho
xn  2 hoặc xn  1 .

59
Thật vậy, giả sử xn có giới hạn hữu hạn là b và xn  2,1 với mọi n.

Khi đó b  2 hoặc b  1 . Giả sử b  2 thì tồn tại N0 sao cho xn nằm trong lân cận 2 với mọi
n  N0. Nhưng nếu xn  2   thì xn 1  2  4   2  xn , suy ra dãy  xn  tăng kể từ N0 và
không thể dần về 2. Nếu b  1 kể từ n  N 0 nào đó xn thuộc lân cận 1. Ta có:

xn  2  xn  2  (2  xn2 ) 2  xn  (2  xn  xn2 )( xn2  x1  1)

Tại lân cận 1 thì xn2  x1  1  0 . Vì nếu xn  1 thì xn 1  1 (và ngược lại xn  1 thì xn  1 , chúng
ta đang xét trong lân cận điểm 1) nên có thể giả sử xn  1 .

Khi đó 2  xn  xn2  0 suy ra xn 2  xn . Tiếp tục như vậy, suy ra 1  xn  xn  2  ...  xn  2 k  ... ,
mâu thuẫn với giả thiết b  1 . Do đó, điều giả sử là sai, tức là dãy số chỉ có giới hạn khi tồn tại n
sao cho xn  2 hoặc xn  1 .

Sau khi thu được kết quả này, ta sử dụng hàm ngược f 1 ( x)   2  x để xây dựng tất cả các
giá trị a thoả mãn điều kiện đầu bài.

Bài 4. (Đề dự bị VMO 2008)

Cho số thực a và dãy số thực {xn } xác định bởi:

x1  a, xn 1  ln(3  cos xn  sin xn )  2008 với mọi n  0,1, 2,...

Chứng minh rằng dãy số  xn  có giới hạn hữu hạn khi n tiến đến dương vô cùng.

Lời giải.

Đặt f ( x )  ln(3  cos x  sin x )  2008 thì f  ( x ) 


cos x  sin x
.
3  sin x  cos x

Từ đó, sử dụng đánh giá | cos x  sin x | 2 , | sin x  cos x | 2 ta suy ra

2
f ( x )   q 1
3 2

Áp dụng dụng định lý Lagrange cho x, y thuộc  , do hàm f ( x) liên tục trên  nên tồn tại z
thuộc  sao cho: f ( x )  f ( y )  f ( z )( x  y )

Từ đó suy ra f ( x )  f ( y )  q. x  y với mọi x, y thuộc  .


60
Gọi l là nghiệm của phương trình f ( x )  x hay f (l )  l .

Ta có f (un )  f (l )  q un  l  q f (un1 )  f (l )  q 2 un1  l ...  q n1 u1  l .

Do q n1  0 khi n   . Theo nguyên lí kẹp thì khi n   , dãy đã cho có giới hạn là l .

Ta có đpcm.

Bài 5. (VMO 2000, bảng A và B)

Cho dãy số  xn  xác định như sau

x0  0, xn 1  c  c  xn , n  0,1, 2,...

Tìm tất cả các giá trị của c để với mọi giá trị x0  (0, c) , xn xác định với mọi n và tồn tại giới hạn
hữu hạn lim xn .

Lời giải.

Để x1 tồn tại thì ta thì c  c  xn  0 với mọi x0  (0, c )  c(c  1)  x0 với mọi x0  (0, c) , suy
ra c  2 . Với c  2 thì 0  x1  c .

Nếu 0  xn  c thì c  c  xn  c  2c  0 , suy ra xn 1 tồn tại và ta cũng có 0  xn 1  c .

Đặt f ( x )  c  c  x thì f ( x )  
1
c x c c x .
4

Với mọi x  (0, c ) ta có (c  x )(c  c  x )  c(c  c  c )  2(2  2  2 ) 


1
.
4

Từ đó suy ra f ( x )  q  1 với mọi x  (0, c ) , tức f ( x) là hàm số co trên (0, c ) , suy ra dãy
số đã cho hội tụ.

Do đó, tất cả các giá trị c cần tìm là c  2 .

Bài 6.

Cho dãy số {xn } xác định bởi x1  (1, 2) và xn 1  1  xn 


xn2
.
2

Chứng minh rằng {xn } có giới hạn hữu hạn khi n dần đến vô cực. Tìm giới hạn đó.

61
Lời giải.

Giả sử xn có giới hạn là a thì a  1  a   a  2 . Ta sẽ dùng định nghĩa để chứng minh


a2
2

lim xn  2 . Ta có xn1  2  1  xn 
xn2 2  xn 1
 2  xn  2 .
2 2

Tiếp theo ta có thể chứng minh bằng quy nạp rằng 1  xn 


3
với mọi n  2,3, 4,...
2
1
2
2 2  xn 1 2  q  1 . Suy ra x  2  q x  2 , n .
Từ đó, suy ra   n1 n
2 2 2

Bằng quy nạp, ta chứng minh được xn1  2  q n x1  2 , n

Vì 0  q  1 nên với n đủ lớn thì q n nhỏ tùy ý, suy ra lim xn  2.

Bài 7. (VMO 1998, bảng A)

 x1  a  1

Cho dãy số xác định bởi   xn2 
 xn1  1  ln  
 1  ln x  , n  1, 2,3,...
n

Chứng minh rằng dãy này có giới hạn hữu hạn không phụ thuộc vào giá trị của a .

Lời giải.

-Nếu a  1 thì xn  1, n hay lim xn  1 .

-Nếu a  1 thì bằng quy nạp, ta chứng minh được rằng xn  1 , tức là chỉ cần chứng minh

xn2  1  ln xn , xn  1 . Bất đẳng thức này có thể chứng minh dễ dàng bằng khảo sát hàm số.

Tiếp theo, ta sẽ chứng minh rằng xn1  xn bằng cách xét hàm số sau

 x 2 
f ( x)  x 1 ln  , x  1
1  ln x 

x 1  x ln x  2ln x
Ta có f ( x )  . Lại xét hàm số g ( x )  x 1  x ln x  2ln x, x  1 , ta có
x (1  ln x )

62
 1
g ( x )  2 1   ln x  0, x  1
 x 

Suy ra f ( x )  0, x  1 hay f ( x ) là hàm đồng biến trên 1; .

 xn2 
Từ đó dễ dàng có được xn1  xn  1  ln  
1  ln x   xn  0  xn  xn 1
.
n

Dãy này giảm và bị chặn dưới bởi 1 nên hội tụ.

 l 2 
Giả sử lim xn  l  1 , ta có l 1 ln    0 . Theo phần chứng minh trên thì dễ thấy rằng
1  ln l 
phương trình này không có nghiệm l  1 và l  1 nghiệm đúng phương trình nên giới hạn cần
tìm của dãy đã cho là l  1 .

Bài 8. (Việt Nam TST 1993)

Dãy số {an } được xác định bởi a1  1 và an 1  an 


1
, n  1, 2,3,...
an

an
Hãy tìm tất cả các số thực  để dãy số có giới hạn hữu hạn khác 0.
n

Lời giải.

Trước hết ta chứng minh an dần tới vô cùng khi n dần tới vô cùng. Thật vật, ta có

an21  a n2  2 a n   a n2  2 (do an  1 với mọi n)


1
an
Từ đó a n21  1  2n suy ra điều phải chứng minh. Trở lại bài toán, xét biểu thức
 
3/ 2
  
1   /  1 
3/ 2

an  an  an 
1        
   3/ 2 
3/2 3/ 2 3/ 2
 a  1 3/ 2 
1
 an   an    an 
n

thì xn   khi n   . Do đó
1
Đặt xn 
an3/ 2

(1  x n )  1 (1  x)  1 3
3 3

 a )  lim  lim 
3 3 2 2

n 1
2 2
lim( a
x 0
n
xn x 2

63
3

 . Với   suy ra giới hạn bằng vô cực, với   suy ra giới hạn
2
a 3 3 3
Từ đó suy ra lim n

n 2 2 2
bằng 0.

Vậy  
3
là giá trị duy nhất thoả mãn yêu cầu bài toán.
2

(n  1) xn1  xn
Bài 9.
Xét dãy số  xn  cho bởi công thức xn 2  .
n
Chứng minh rằng với mọi giá trị ban đầu x1 , x2 dãy số đã cho hội tụ. Tìm giới hạn của dãy như
một hàm số theo x1 , x2 .

Lời giải.
Ta có từ công thức của dãy số, ta có
( xn 1  xn ) ( xn  xn 1 ) (1) n ( x2  x1 )
xn  2  xn 1     ... 
n(n  1)
.
n n!
Từ đó suy ra
xn  2  ( xn  2  xn 1 )  ( xn 1  xn )  ...  ( x2  x1 )  x1  x1  ( x2  x1 ).K n ,

(1)n
trong đó K n  1    ...  , ta thấy K n  , n   .
1 1 1
1! 2! n! e
( x2  x1 )
Từ đây suy ra dãy số có giới hạn và giới hạn đó bằng x1  .
e

Dãy số an  được xác định bởi a1  0 , a2  0 và an 1 


Bài 10.
2
an  an 1
.

Chứng minh rằng dãy số an  hội tụ với mọi giá trị của a1 , a2 và giới hạn của dãy không phụ
thuộc vào hai giá trị này.
Lời giải.
Xét hai dãy M n  max an , an 1 , an 2 , an 3  và mn  min an , an 1 , an  2 , an 3  . Ta chứng minh M n là
dãy số giảm và mn là dãy số tăng.

Thật vậy, ta sẽ chứng minh an  4  max an 1 , an 3  .

Từ đây suy ra M n 1  an 1 hoặc an  2 hoặc an 3 và khi đó M n  max an , an 1 , an 2 , an 3   M n 1 .

64
Thật vậy, nếu an  4  an 3 thì  an 3 suy ra 2  (an 3  an  2 )an 3 .
2
an  3  an  2

Khi đó an 1   an  2    an 2  an  4  2.  an  2  an  4  an  4
2 2 2 an  2
an  3 an  3 an 2  an 3 ( an  3  an  2 ) an  3
suy ra đpcm.
Ta đã chứng minh được M n giảm. Tương tự mn tăng. Hai dãy số này đều bị chặn nên hội tụ.

Suy ra dãy an  hội tụ và lim an  1.


Cuối cùng, ta chứng minh được hai giới hạn bằng nhau.

Bài 11.

Cho un  là dãy bị chặn thỏa mãn: 2an 2  an  an 1 , n . Chứng minh rằng dãy an  này hội tụ.

Lời giải.

Đặt An  max{an , an 1} . Ta sẽ chứng minh dãy này hội tụ. Thật vậy

an  an 1
an  2   An  An 1  max{an 1 , an  2 }  max{ An 1 , An }  An  2 .
2

Do dãy an  bị chặn nên dãy  An  cũng bị chặn, đồng thời theo nhận xét trên thì dãy  An  giảm
nên nó hội tụ. Đặt lim An   .

 
Với mọi   0 , tồn tại N nguyên dương sao cho với mọi: n  N thì:    An    .
3 3


Theo định nghĩa của  An  , suy ra: an  An    .
3

  
- Nếu an    thì suy ra:    an    ,   0  lim an   .

 
3 3 3
- Nếu: an    thì theo định nghĩa của  An  , ta đợc an 1    . Suy ra:

   
3 3

an  2an 1  an 1  2              , tức là:


 3  3

    an     ,   0  lim an   .

Vậy trong mọi trường hợp, dãy đã cho đều có giới hạn.

65
Bài 12. (VMO 2001, bảng A)
Với mỗi cặp số thực (a, b) , xét dãy số thực sau

x0  a, xn1  xn  b sin xn , n  0,1, 2,...

a) Cho b  1 . Chứng minh rằng dãy này có giới hạn hữu hạn, tìm giới hạn đó.

b) Chứng minh rằng với b  2 thì luôn tồn tại a sao cho dãy  xn  tương ứng không có giới hạn.

Lời giải.

a) Với b  1 , ta có dãy số tương ứng x0  a, xn1  xn  sin xn , n  0,1, 2,... Ta xét các trường hợp

-Nếu a  k , k   thì xn  a, n   , tức là dãy này có giới hạn.

-Nếu a  k , k   . Xét hàm số f ( x )  x  sin x, x   thì xn1  f ( xn ), n  0,1, 2,... và

f ( x )  1  cos x  0 nên hàm này đồng biến và dãy đã cho đơn điệu. Ta có 2 khả năng:

+Nếu a   2k ;(2k  1)  , k   .

Khi đó, sin a  0 và suy ra dãy này đơn điệu tăng.

Bằng quy nạp theo n, ta chứng minh được rằng xn   2k ;(2k 1)  , n .

Suy ra dãy này tăng và bị chặn trên nên có giới hạn, đặt đó là l thì chuyển qua giới hạn, ta có

2k   a  l  (2k  1),sin l  0

Do đó l  (2k 1) .

+Nếu a  (2k 1); 2k   , k   . Khi đó, sin a  0 và suy ra dãy này đơn điệu giảm. Chứng
minh tương tự phần trên, dãy này giảm và bị chặn dưới nên có giới hạn. Ta tính được giới hạn đó
chính là (2k 1) .

Suy ra, với b  1 thì dãy đã cho luôn có giới hạn và giới hạn đó được xác định bởi công thức

  a   a 
lim xn  2    sign    
  2   2 

Với  x ,  x , sign( x ) lần lượt là hàm phần nguyên, hàm phần lẻ và hàm dấu của x.

66
sin x
b) Xét hàm số g ( x )  , x  (0; ] . Ta thấy đây là hàm liên tục và g ( )  0, lim  1 nên từ
x x0

2
điều kiện 0   1 do b  2 nên tồn tại a0  0;   sao cho 2a0  b sin a0 .
b

Dễ thấy rằng khi đó dãy ( xn ) tuần hoàn với chu kì nhỏ nhất là 2 nên không có giới hạn hữu hạn.

Ta có đpcm.

Bài 13.

Dãy số un  xác định bởi u1  b, un1  un2  (1 2a )un  a 2 , n  1, 2,3,...

Tìm tất cả các giá trị của a, b sao cho dãy này có giới hạn hữu hạn. Tìm giới hạn đó.

Lời giải.

Ta thấy rằng un1  un  (un  a )2 nên nếu dãy đã cho có giới hạn là L thì

L  L  ( L  a )2  L  a .

Dễ thấy dãy đã cho tăng nên nếu tồn tại số i nào đó mà ui  a  u j  a, j  i , tức là dãy không
hội tụ.

Nếu tồn tại n sao cho un2  (1 2a)un  a 2  a thì a 1  un  a  a 1  b  a (vì nếu ngược
lại thì lại quay về trường hợp trên và dãy không hội tụ).

Ngược lại, ta xét a 1  b  a . Ta có un1  un  ...  u1  b  a 1 và

un1  un2  (un  a )2  un  un  a  un  a  un  a

Khi đó, a 1  un  a, n và dãy này tăng nên có giới hạn. Giới hạn đó chính là lim un  a .

Bài 14. (VMO 2004, bảng A)

Xét dãy số thực ( xn ), n  1, 2,3,... xác định bởi

(2  cos 2 ) xn  cos 2 
x1  1, xn1  , n  1, 2, 3,...
(2  2cos 2)  2  cos 2

trong đó  là tham số thực.

67
n
1
Hãy xác định tất cả các giá trị của  để dãy số  yn  với yn   , n  1, 2,3,... có giới
k 1 2 xk 1
hạn hữu hạn. Tìm giới hạn tương ứng trong các trường hợp đó.

Lời giải.

Vì x1  1  0, 2  cos 2  0, cos 2   0, 2  2 cos   0, 2  cos 2  0 nên từ công thức đã cho,


3(2 xn  1)
ta thấy rằng xn  0, n . Ta có 2 xn 1  1  . Suy ra
(2  2cos 2 ) xn  2  cos 2

1 (1 cos 2 )(2 xn 1)  1 1  1 


   2 sin 2   
2 xn 1  1 3(2 xn  1) 3 2 xn  1

1 1 
 sin 2  . Đây là một cấp số nhân có công bội là và số
1 1
Do đó,  sin 2   
2 xn 1  1 
3  2 xn 1   3
1
hạng đầu là  sin 2  nên ta có thể xác định được tất cả số hạng của dãy. Từ đó có được
3
n
1 1 3sin 2   1
yn    1 n   n sin , n  1
2

k 1 2 xk  1 2 3

1
Vì dãy số n
hội tụ nên  yn  hội tụ khi n sin 2  hội tụ hay sin 2   0    k , k   .
3

1
Khi đó, giới hạn tương ứng của dãy yn là .
2

Bài 15. (VMO 2005, bảng A)

 x1  a
Xét dãy số thực ( xn ) được xác định bởi công thức  , n  1, 2,3,...
 xn1  3xn3  7 xn2  5 xn

Hãy tìm tất cả các giá trị của a sao cho dãy số  xn  có giới hạn hữu hạn và tìm giới hạn của dãy
tương ứng trong các trường hợp đó.

Lời giải.

Xét hàm số f ( x )  3 x3  7 x 2  5 x, x   .

Dãy số đã cho chính là x1  a, xn1  f ( xn ), n  1, 2,3,...

68
Ta có f ( x )  9 x 2 14 x  5  (9 x  5)( x 1) . Ta có bảng biến thiên sau

 
5
x 1
9

y + 0  0 +

275

y 243

 1

4
Ta cũng có f ( x)  x  x( x 1)(4 x  3) nên f ( x )  x có ba nghiệm là x  0, x  1, x  .
3

4  4 4
Dễ thấy rằng x  0 thì f ( x )  x và x  thì f ( x )  x . Đồng thời f (0)  0, f    nên từ
3  3  3
bảng biến thiên ở trên, ta thấy rằng:

- Với x  ; 0 thì ta luôn có f ( x)  (;0) .

 4  4
- Với x  0;  thì ta luôn có f ( x )  0;  .
 3   3 

4  4 
- Với x   ;  thì ta luôn có f ( x )   ;  .
 3   3 

69
Ta xét các trường hợp sau:

-Nếu a  0 thì xn  0, n với mọi n  1 . Ta có x2  f ( x1 )  x1 và hàm f ( x ) đồng biến trên


miền này nên dãy đã cho đơn điệu giảm, tức là nếu dãy này hội tụ về l thì l  0 , không thể là
nghiệm của phương trình f ( x )  x đã nêu nên trong trường hợp này, dãy không hội tụ.

4 4
-Nếu a  thì xn  , n . Ta có x2  f ( x1 )  x1 và f ( x ) đồng biến trên miền này nên dãy đã
3 3
4
cho đơn điệu tăng, tức là nếu dãy này hội tụ về l thì l  , cũng không thể là nghiệm của
3
phương trình f ( x )  x đã nêu nên trong trường hợp này, dãy không hội tụ.

-Nếu a  0 thì xn  0, n nên thỏa mãn đề bài.


4 4
-Nếu a  thì xn  , n nên cũng thỏa mãn đề bài.
3 3
4  4
-Nếu 0  a  thì suy ra xn  0;  , n . Ta có hai trường hợp sau:
3  3 
 4
+ Nếu a  1;  thì ta có
 3 
f (a) 1  3a 3  7 a 2  5a 1  (a 1) 2 (3a 1)  0
4 4 4
f (a)   3a 3  7 a 2  5a   (a  )(3 x 2  3 x 1)  0
3 3 3
 4
Do đó, bằng quy nạp, ta chứng minh được n  , xn  1;  .
 3 

70
Hàm số f ( x ) trên miền này đồng biến và f (a)  a  3a 3  7 a 2  4a  (a 1)(3a  4)  0 nên
dãy số này giảm và bị chặn dưới. Do đó, dãy số  xn  có giới hạn. Dễ thấy rằng giới hạn đó phải
 4
thuộc tập nghiệm của phương trình f ( x )  x và thuộc miền 1;  , suy ra giới hạn là 1.
 3 
+Nếu a  0;1 thì có hai trường hợp xảy ra:
*Nếu như n   mà xn  0;1 thì f ( x)  x, x  0;1 nên với n tùy ý thì xn1  f ( xn )  xn ,
tức là dãy này đơn điệu tăng. Hơn nữa, nó bị chặn trên nên có giới hạn. Lập luận tương tự trên,
giới hạn đó cũng chính là 1.
 4
*Nếu như tồn tại một số số hạng của dãy thuộc miền xn  0;  nhưng không thuộc an  0;1
 3 
thì gọi n0 là chỉ số nhỏ nhất sao cho xn0  0;1 , rõ ràng n0  1 vì a  x1  0;1 . Khi đó, ta có
 5  275
an0 1  0;1 nên xn0  max f ( xn0 1 )  f    và xn0  min f ( xn0 1 )  f (0)  0 .
 9  243
 275   275   4 
Do đó, xn0  0;      1;  1; . Suy ra từ số hạng này trở đi thì tất
 243   3 
và x 0;1 nên x
 243  n0 n0

 4
cả các số hạng của dãy đều thuộc 1;  và trường hợp này đã được chứng minh ở phần trên.
 3 
Giới hạn của dãy số trong trường hợp này vẫn là 1.
 4
Tóm lại, dãy đã cho hội tụ khi và chỉ khi a   0;  , cụ thể là:
 3 
+ Với a  0 thì dãy hội tụ về 0.
4 4
+ Với a  thì dãy hội tụ về .
3 3
4
+ Với 0  a  thì dãy hội tụ về 1.
3
Bài 16.
6  6an
Cho dãy số xác định bởi a1  0, an1  , n  1, 2,3,...
7  an
Chứng minh rằng dãy này luôn hội tụ và tìm giới hạn của nó.
Lời giải.
 6 
Ta thấy rằng an 1  6 1  nên bằng quy nạp, ta có thể chứng minh được rằng an  2, n
 an  7 
nếu a1  2 và an  2, n nếu a1  2 .

71
6x  6 36
Xét hàm số f ( x)  , x  0 . Ta có f ( x )   0, x  0 và
x7 ( x  7)2
( x  3)( x  2)
f ( x)  x   . Do đó:
x7

-Nếu 0  a1  2 thì dãy này tăng và bị chặn trên bởi 2 nên có giới hạn.

-Nếu a1  2 thì dãy này giảm và bị chặn dưới bởi 2 nên cũng có giới hạn.

-Nếu a1  2 thì đây là dãy hằng.

Tóm lại, trong mọi trường hợp thì dãy đã cho đều hội tụ và giới hạn của nó là nghiệm dương của
phương trình f ( x)  x  x  2 .

Bài 17.
2 x3  3
Cho hàm số thực f ( x) 
3( x 2  1)
.

1. Chứng minh rằng tồn tại hàm số g ( x) liên tục trên  sao cho f ( g ( x ))  x và g ( x)  x với
mọi số thực x.
2. Chứng minh rằng tồn tại số thực a  1 để dãy {un }, n  1, 2,3,... được xác định bởi
u0  a, un 1  f (un ), n  1, 2,3,... tuần hoàn với chu kỳ dương nhỏ nhất bằng 2011.

Lời giải.

2 x( x 3  3x  3)
1. Hàm số f ( x ) xác định trên (; 1)  (1;1)  (1; ) . Ta có f ( x) 
3( x 2  1)2
.

Ta thấy rằng x 3  3 x  3  ( x  1)2 ( x  2)  1  0, x  1 nên hàm số đã cho đồng biến trên (1; ) .

Hơn nữa, lim f ( x )  , lim f ( x)   nên với tập xác định là (1; ) thì tập giá trị của f ( x )
x 1 x 

là (; ) , mà f ( x ) là hàm liên tục và đồng biến trên (1; ) nên nó chính là một song ánh từ
(1; ) vào (; ) .

Do đó, tồn tại hàm số ngược g ( x) của f ( x ) có tập xác định là  và tập giá trị là (1; ) sao
cho f ( g ( x ))  x, x   .

Ta cần chứng minh rằng g ( x )  x, x   . Thật vậy:

- Với x  1 thì g ( x )  1  x .

72
x3  3x  3
- Với x  1 thì f ( x)  x    0, x  1  x  f ( x), x  1  f ( g ( x ))  f ( x), x  1 ,
3( x 2  1)
do tính đồng biến của f ( x ) trên (1; ) nên g ( x )  x, x  1 .

Vậy hàm số g ( x) thỏa mãn đề bài. Ta có đpcm.

2. Đặt g n ( x )  g ( g ...g ( x))....) , n  * và u0  a  g 2010 ( x0 )  1 với x0   sẽ được xác định sau.


 
n

Ta có u0  g 2010 ( x0 )  1  u1  f (u0 )  f ( g 2010 ( x0 ))  g 2009 ( x0 )  1 .

Tiếp tục quá trình này, ta dễ dàng chứng minh được u2010  x0 đồng thời vì g ( x )  x, x   nên
u0  u1  u2  ...  u2010 .

Do đó, nếu chọn được x0 sao cho x0  1 và u2011  f ( x0 )  g n ( x0 ) thì dãy đã cho có chu kì
dương nhỏ nhất là 2011.
Ta chỉ cần chứng minh tồn tại x0 thỏa mãn điều kiện nêu trên.

Thật vậy, xét hàm h( x)  f ( x)  g 2010 ( x) trên (1;0] , dễ thấy h( x) là hàm liên tục trên nửa
khoảng này và h(0)  f (0)  g 2010 (0)  1  g 2010 (0)  0 , lim h( x)  lim f ( x)  g 2010 (1)  
x 1 x 1

nên phương trình h( x)  0 có nghiệm x0 trên (1;0] .

Vậy tồn tại giá trị a  1 thỏa mãn đề bài nên ta có đpcm.

Bài 18. (Việt Nam TST 1996)


Hãy tìm tất cả các số thực a và b để dãy số sau có giới hạn hữu hạn
a
x0  b  0, xn1  , n  0,1, 2,...
1  xn2

Lời giải.

Ta xét các trường hợp sau :

Nếu a  0 thì hiển nhiên xi  0, i  0 nên dãy đã cho hội tụ.

a 2ax
Nếu a  0 thì kí hiệu f ( x )  và g ( x )  f  f ( x) thì ta có f ( x )    0 nên
1 x 2
(1  x 2 )2
f ( x ) là hàm giảm trên 0; và hàm g ( x) tăng trên 0; .

73
Xét dãy  x2n  và đặt un  x2 n , n  0,1, 2,... thì un1  g (un ) . Do g ( x) tăng nên un  là dãy đơn
điệu (tăng hoặc giảm). Mặt khác, dễ thấy rằng 0  un  a nên dãy này bị chặn và luôn tồn tại
giới hạn lim un  lim x2 n  l . Giới hạn này chính là nghiệm của phương trình g (l )  l .

Ta có x2 n 1  f ( x2 n ) nên nếu dãy  x2n  có giới hạn thì dãy  x2 n 1  cũng có giới hạn.

Ta sẽ chứng minh rằng nếu 0  a  2 thì hai dãy này có cùng giới hạn, tức là  xn  có giới hạn.
Thật vậy, giả sử f (l )  v hay a  v  vl 2 . Ta cũng có f  f (l )  f (v )  f (v )  g (l )  l hay
a  l  lv 2 .

Trừ tương ứng hai vế của các đẳng thức trên, ta được (v  l )(vl 1)  0 .

Nếu v  l thì vl  1 , suy ra v  l  a , tức là v, l là nghiệm của phương trình x 2  ax  1  0 .

Vì a  2 nên hoặc là phương trình này vô nghiệm, hoặc là với a  2 thì nó có nghiệm v  l  1 ,
trái với điều giả sử. Do đó, v  l hay lim x2 n1  lim x2 n  f (l )  l .

Do đó, trong trường hợp này, giới hạn của dãy  xn  tồn tại.

Ta sẽ chứng minh trong trường hợp a  2 thì dãy đã cho hội tụ khi và chỉ khi a  x03  x0 . Thật
a x0  x03
vậy, nếu a  x03  x0 thì x1    x0 . Tương tự, ta chứng minh được dãy này
1  x02 1  x02
không đổi nên nó có giới hạn.

Giả sử a  x0  x03 và tồn tại giới hạn lim xn  k  0 thì chuyển qua giới hạn, ta có f (k )  k
hay a  k  k 3 . Do đó, k chính là nghiệm duy nhất của phương trình x 3  x  a . Ta có

(1  x 2 )2 (a  x)  a 2 x ( x 2  x  a)( x 2  ax 1)
g ( x)  x  
a 2  (1  x 2 )2 a 2  (1  x 2 )2

Vì a  2 nên phương trình x 2  ax  1  0 có hai nghiệm phân biệt là ,  . Nếu giả sử   


thì   k   . Thật vậy, vì a  2 nên k  1 , do đó, a  a 2 k  0 . Thay a  k  k 3 vào, ta được

k 3  k  ak 2  0    k   .

Dấu của g ( x)  x chính là dấu của ( x  k )( x  )( x   ) . Từ bảng xét dấu của biểu thức này,
ta được

74
Nếu xét dãy un  x2 n tương tư ở trên thì nó đơn điệu và lim un  k nên tồn tại n  n0 sao cho
  un0   . Nếu   un0  k thì un0 1  g (un0 )  un0 , suy ra un  là dãy giảm kể từ un0 và
un0  k nên không thể có lim un  k . Tương tự nếu k  un0   . Suy ra chỉ có thể là un0  k .

Ta cũng có, nếu i : xi  k , i  1  xi1  k vì

a
xi   k  k 1  xi21   a  k  k 3  xi21  k 2  xi1  k
1  xi21

Suy ra, nếu un0  k thì u0  k , mà a  k  k 3 nên a  x0  x03 , mâu thuẫn với điều giả sử.

a a
Nếu a  0 , đặt xn   yn  yn 1   và yn  0, n  1 .
1  xn 1  yn2
2

Do đó,  xn  hội tụ khi và chỉ khi  yn  hội tụ. Tương tự như lập luận ở trên, dãy này có hội tụ
khi 0  a  2 hoặc a  y1  y13 . Hơn nữa, a  y1 1  y02   y1 1  x02   x0  y1 hay
a  x0  x03 , tương tự điều kiện trên.

Vậy dãy đã cho có giới hạn hữu hạn khi và chỉ khi a  2 hoặc a  x0  x03 .

Bài 19.

x 
2

Chứng minh rằng dãy số xác định bởi xn1  xn   n  , n  1 với x1  0;1 có giới hạn hữu hạn.
 n 

Lời giải.

x22
Ta có x2  x1  x12  2 x1 và x3  x2   2 x1  x12  3 x1 ,… Bằng quy nạp, ta chứng minh được
4
rằng xn  nx1 , n  1 . Ta luôn có thể chọn được số m sao cho xm  m 1 vì

1
mx1  m 1  m(1 x1 )  1  m 
1 x1

Các giá trị m như thế là tồn tại và xm  mx1  m 1 thỏa điều kiện.

1 1 x  xn xn2 / n 2 xn 1 1 1 1
Hơn nữa,   n 1    2   .
xn xn 1 xn xn1 xn xn1 xn1n 2
n n(n 1) n 1 n

75
n1 
1  n1  1 
   
1 1 1 1 1 1 1
Do đó    
     .
 
xm xn i0  xmi xmi1  i0  m  i 1 m  i  m 1 n 1 m 1

1 1 1 (m 1) xm
Suy ra    0  xn  .
xn xm m 1 (m 1)  xm

Vế phải của bất đẳng thức này không phụ thuộc vào n nên dãy này bị chặn trên. Hơn nữa, dãy đã
cho cũng tăng thực sự nên nó có giới hạn hữu hạn.

Bài 20.
Cho dãy số dương xác định bởi 8 x2  7 x1  x17  8 và

xk81  xk8
xk 1 xk1  x  7 7 , k  2, 3, 4,...
2
k
xk xk 1
Xác định một số thực dương a sao cho nếu x1  a thì dãy này đơn điệu giảm và nếu 0  x1  a
thì dãy này không đơn điệu.
Lời giải.
Từ công thức dãy đã cho, ta có
xk2 x8  x 8 8x9 x8  x 8
xk 1   k71 8 k  8 xk 1 xk7  k  8 k 18 k
xk 1 xk xk 1 xk 1 xk 1
xk8 8 x8
 8 xk 1 xk7  7 xk8  8 xk  7 xk 1   8  8 k
xk 1 xk 1
xk8 
 (8 xk 1  7 xk ) x  8  8 8 xk  7 xk 1  xk71  8
7
k
xk 1

Từ điều kiện 8 x2  7 x1  x17  8 và bằng quy nạp, ta chứng minh được

1 7 xk
(8 xk 1  7 xk ) xk7  8  xk 1   , k  1, 2, 3,...
xk7 8
1 7x
Xét hàm số f ( x )   , x  0 . Ta thấy rằng f ( x)  x  x 8  8  x  8 8 .
x7 8
Do đó, f ( x)  x, x  8 8 và f ( x)  x, x  8 8 .

Ta sẽ chứng minh rằng a  8 8 là giá trị cần tìm.

76
Thật vậy, nếu x1  a thì x2  f ( x1 )  x1 và bằng quy nạp, ta chứng minh được rằng xn  8 8, n
7 7 7  8
và vì f ( x )     1 8   0, x  8 8 nên đây là hàm đồng biến và dãy đã cho đơn
x 8
8 8  x 
điệu giảm.
Nếu x1  a thì x2  f ( x1 )  x1 và bằng quy nạp, ta chứng minh được rằng xn  8 8, n và ta
cũng có f ( x )  0, x  8 8 nên đây là hàm nghịch biến, suy ra dãy đã cho không đơn điệu.

Vậy giá trị cần tìm là a  8 8 .


Bài 21.

 1 1 1
Cho dãy số  1; ; ;...;  với n là số nguyên dương cho trước. Ta xét hai hoán vị sau của dãy
 2 3 n 
trên là a1 , a2 , a3 ,..., an  và b1 , b2 , b3 ,..., bn  thỏa mãn điều kiện

a1  b1  a2  b2  ...  an  bn

4
a) Chứng minh rằng ak  bk  , k  1, n .
k
b) Chứng minh rằng với mỗi c  1 thì luôn tồn tại ít nhất một số tự nhiên n sao cho có hai hoán
4c
vị thỏa mãn điều kiện trên và bất đẳng thức ak  bk  đúng đối với ít nhất 1 giá trị k  1, n .
k

Lời giải.

a) Với mỗi k, ta chia các cặp a1 , b1  , a2 , b2  ,..., ak , bk  thành ba loại sau:

- Loại 1 gồm r cặp với ai  bi .

- Loại 2 gồm s cặp với ai  bi .

- Loại 3 gồm t cặp với ai  bi .

Khi đó, r  s  t  k và do tính đối xứng giữa ai , bi nên ta có thể giả sử r  t .

Ta xét hai trường hợp sau:

-Nếu s  0 thì ta có r  s cặp loại 1 và 2 là ai1 , bi1  ,  ai2 , bi2  ,..., air s , bir s  .

77
 1 1 1
Các số ai j ,1  j  r  s lấy r  s giá trị khác nhau trong tập hợp 
1; ; ;...;  nên nếu đặt
 2 3 n 


a*  min ai j :1  j  r  s   1
rs

2 4 4 4
Gọi a* , b*  là cặp số tương ứng thì ak  bk  a*  b*  2a*     .
r  s 2r  2 s r  s  t k

-Nếu s  0 thì ta xét r cặp loại 1 là ai1 , bi1  ,  ai2 , bi2  ,..., air , bir  .

 1

Ta cũng đặt a*  min ai j :1  j  r  s thì a*  , b*  a* do a* , b*  là một cặp loại 1.
r

2 4 4 4
Vì vậy, ak  bk  a*  b*  2a*     .
r 2r r  t k

Trong cả hai trường hợp, ta luôn có đpcm.

b) Xét n  2m, m  * thì ta xét hai hoán vị sau

 1 1 1 1 1 
a1 , a2 , a3 ,..., an   1, , , ,..., ,  , b1 , b2 , b3 ,..., bn    a2 , a1 , a4 , a3 ,..., a2 m , a2 m1  .
 2 m 2 2 m 1 m m 1

Có thể thấy rằng hai hoán vị này thỏa mãn điều kiện ban đầu và

1 1 2m 1
a2 m  b2 m    .
m  1 m m(m 1)

4c 2  (m  1)c
Do đó, với c  0 thì  a2 m  b2 m   .
2m 2m

Với c dương tùy ý thì ta luôn có thể chọn m sao cho biểu thức trên âm. Từ đó, ta có đpcm.

Bài 22.

Cho p là một số nguyên tố , xét dãy số (an ) thỏa mãn

a0  0, a1  1, ak  2  2a k 1  pak k  0,1, 2,...

Xác định tất cả giá trị của p để 1 là một phần tử của dãy (an ) .
Lời giải.

78
Nếu p  2 , khi đó ak  2  2a k 1  2a k với mọi k  0,1, 2,... , do đó ak là số chẵn với mọi k  2
nên không thể có một phần tử nhận giá trị là 1 . Suy ra với p  2  ( p, 2)  1 .
Giả sử tồn tại m  2 sao cho am  1 , theo đề ta có ak 1  2a k  pak 1  2a k (mod p), k  2,3, 4,...

Suy ra 1  am  2m1  a1  2m 1 (mod p) . (*)


Ta cũng có
ak 1  2a k  a k 1  ( p  1)ak 1  2a k  ak 1 (mod p  1)  ak 1  ak  ak  ak 1 (mod p  1), k  2,3, 4,...
Suy ra ak 1  (k  1)(a1  a0 )  a0  k  1(mod p  1) .

Do đó 1  am  m(mod p  1)  m  1  0(mod p  1) . (**)

Do ( p, 2)  1 nên theo định lý Fermat, ta có 2 p 1  1(mod p )  2m 1  1(mod p )

Kết hợp với (*), ta được 4  2m 1  1(mod p)  5  0(mod p)  p  5 .


Với p  5 , ta thấy a3  1 , thỏa mãn đề bài.
Vậy tất cả giá trị của p thỏa yêu cầu bài toán là p  5 .
Bài 23.
k  un
Cho dãy số un  xác định bởi u1  a, un1  , n  0 . Biết rằng u1  u13 .
1 un
Hãy tìm tất cả các giá trị của k.
Lời giải.
un
k
k  un u k .
Ta có un1   n1 
1 un k 1 k  un
k
a    
Đặt k  tan ,  tan  0    ,      . Bằng quy nạp, ta chứng minh được rằng
k  2 2 2
un
 tan (n 1)    , n
k
Do đó, u1  u13  tan(12   )  tan   12  m, m   .

Vì 0    nên m  1, 2,3, 4,5 .
2

79

1 cos
 6  2  3  7  4 3 , suy ra tan 2 5  cot 2   7  4 3 .
Ta cũng có tan 2 
12 1  cos  2  3 6 12
6
 1 
Do đó, các giá trị của m cần tìm là m  7  4 3, ,1, 3, 7  4 3 .
 3 
Bài 24. (Việt Nam TST 2009)

Cho đa thức P ( x)  rx3  qx 2  px  1 trong đó p, q, r là các số thực và r  0.

Xét dãy số thực sau

a1  1, a2   p, a3  p  q

2

an 3   p.an  2  q.an 1  r.an , n  0

Chứng minh rằng nếu đa thức P( x ) có một nghiệm thực duy nhất và không có nghiệm bội thì
dãy số an  có vô số số âm.

Lời giải.
Giả sử k là một nghiệm (thực hoặc phức) của đa thức

Q ( x)  x 3  px 2  qx  r , do r  0 nên k  0  k 3  pk 2  qk  r  0 (*)

Theo giả thiết, đa thức P ( x)  rx3  qx 2  px  1 có đúng một nghiệm thực nên nó còn có thêm
1
hai nghiệm phức liên hợp nữa, đồng thời chính là nghiệm của P( x ) do
k
1 1 1 r  qk  pk 2  k 3
P( )  r    q    p    1  0.
3 2
1
k k k k k3

Xét dãy số un  xác định bởi công thức un 1  an 3  ( p  k )an  2  an 1


r
(**)
k
Theo giả thiết thì an 3   pan  2  qan 1  ran , n  0,1, 2,... Ta có:
kq  r
un 1   pan  2  qan 1  ran  ( p  k )an  2  an 1  kan  2  .an 1  ran
r

kq  r
k k
 k (an  2  2 .an 1  an )
r
k k

kq  r
Từ (*) suy ra (kq  r )  pk 2  k 3    p  k , do đó:
k2

80
un 1  k (an  2  ( p  k )an 1  an )  kun , n  0,1, 2,...
r
k
Trong (**), cho n  1 , ta có
pk 2  qk  r k 3
u0  a2  ( p  k )a1  a0  p 2  q  ( p  k ) p      k2
r r
k k k k
Suy ra un  k n  2  an  2  ( p  k )an 1  an  k n  2 , n  0,1, 2,...
r
(***)

Giả sử z là nghiệm phức của phương trình P( x)  0 và  ,  lần lượt là modun và argument của
k

z trong đó:  ,  ,   0 .

Ta có: z   ei   (cos   i sin  ) và P( x )  [ x ] nên: P ( z )  0  P ( z )  0  P ( z )  0 do đó


z   e i   (cos   i sin  ) cũng là nghiệm của P(x).

Từ (***), ta được an  2  ( p  z )an 1  an  ( z )n  2 , an  2  ( p  z )an 1  an  ( z ) n  2 .


r r
z z

Theo công thức Moavre, ta có: z   (cos   i sin  )  z n   n (cos n  i sin n ) nên:


z n 2  z   n  2  cos(n  2)  i sin(n  2)    n  2 cos(n  2)  i sin(n  2)  
n2

z n 2  z 2i. n  2 sin  (n  2)  sin ( n  2) 


n 2

 2i. n  2 sin  (n  2)      n 1.


zz 2i. sin  sin 

Trừ từng vế hai đẳng thức, ta được ( z  z )an 1  r (  )an  ( z )n  2  ( z ) n  2 .


1 1
z z
zz z n 2  ( z ) n 2
 ( z  z )an 1  r ( )an  z n  2  ( z ) n  2  an 1  2 an 

r
(z  z)
sin  (n  2) 
z.z

 an 1  a   n 1 .
 sin 
r
2 n

Do   0 nên xét n0 là một giá trị nguyên dương sao cho


sin  (n0  2)  sin (n0  2) 
 0   n0 1 .  0  an0 1  2 an0  0 .
sin  sin  
r

 0 nên an0 1 , an0 trái dấu với nhau. Do đó, trong hai giá trị này có một số âm.
2
r

sin  (n0  2) 


 0 , mà ứng với mỗi
sin 
Ta thấy khi n tiến tới vô cực, tồn tại vô số giá trị n0 sao cho

giá trị n0 như thế ta lại tìm được một số hạng âm của dãy đã cho, tức là dãy (an) có vô số số âm.
Đây chính là điều phải chứng minh.
81
E – CÁC BÀI TOÁN DÃY SỐ CÓ GỢI Ý GIẢI.

Bài 1. (Việt Nam TST 1985)


Cho dãy số thực ( xn ) được xác định bởi công thức:

x1  , xn 1   3, n  1, 2,3,...
29 xn
10 xn2  1
Chứng minh rằng tồn tại một số thực a thỏa mãn x2 k 1  a  x2 k với mọi số nguyên dương k.

Gợi ý.

Từ cách xác định dãy số, ta có xn  3, n . Ta sẽ tìm giới hạn của dãy số đã cho và chứng minh
rằng giới hạn đó chính là giá trị cần tìm.
x
Xét hàm số f ( x )   3, x  3 .
x 1
2

3( 5  1)
Phương trình f ( x )  x có nghiệm duy nhất là x  .
2
1
Hàm số f  ( x )   0, x  3 nên đây là hàm nghịch biến.
 
3
x 1
2

Đến đây, ta có bài toán quen thuộc là tìm giới hạn của dãy số xác định bởi công thức truy hồi
xn1  f ( xn ) và f ( x ) nghịch biến.

Sử dụng tính chất của hai dãy con có tính đơn điệu ngược nhau là hoàn tất chứng minh.
Bài 2.

Cho n số dương a1 , a2 , a3 ,..., an với a1  1, an  2 và ak  ak 1ak 1 với k  2,3, 4,..., n 1 .

Xác định số lớn nhất của dãy.

Gợi ý. Bằng phản chứng hãy chứng minh rằng max ak  max{a1 , an } .
1k n

Bài 3.

1
Chứng minh rằng dãy số  xn  có tính chất xn  xm  , n  m .
n

Chứng minh rằng dãy này không bị chặn.

82
n
1
Gợi ý. Trước hết, chứng minh rằng dãy an     . Giả sử dãy đã cho bị chặn, tức là tồn
i 1 i

1
tại xn  M , n rồi xét lân cận bán kính cho mỗi điểm.
2n

Bài 4.

Tìm tất cả các số thực a, b sao cho dãy  xn  với x0  a, xn1  1  bxn , n  0,1, 2,... hội tụ.

Gợi ý. Quy nạp tìm công thức tổng quát của n rồi tìm biện luận.

1
Kết quả là b  1, a tùy ý hoặc b  1, a  .
1 b

Bài 5.

xn3  3axn
Cho dãy  xn  xác định bởi x1  0 và xn1  ,a 0.
3xn2  a

Chứng minh rằng dãy này có giới hạn với mọi x1 . Tìm giới hạn đó.

x3  3ax
Gợi ý. Dễ thấy rằng hàm số f ( x)  , a  0 có đạo hàm dương và dãy số đã cho chính là
3x 2  a
xn1  f  xn  , n  1, 2,3,...

Bài 6.


Cho dãy số  yn  xác định bởi y1  x  , yn 1  a sin yn trong đó a  ,x0.
2

Tìm giới hạn của dãy này.


Gợi ý. Xét các trường hợp 1  a  1 thì giới hạn bằng 0, nếu 1  a  thì xét nghiệm l * của
2
phương trình l  a sin l . Các trường hợp khác không có giới hạn.

Bài 7.

Cho dãy số  xn  xác định như sau: xn1  sin xn , n  1, 2,3,... và x1  0;   . Chứng minh rằng
lim nxn  3 .

83
1 1 1
Gợi ý: Chứng minh rằng tồn tại dãy yn  0 sao cho 2
 2   yn . Đánh giá các tổng này
xn xn1 3
để suy ra các hằng số, từ đó tìm được giới hạn.

Bài 8.

Giả sử a1 , a2 , a3 ,... là dãy các số tự nhiên vô hạn không nhỏ hơn 2. Chứng minh rằng từ dãy này
có thể trích ra được dãy con ai1 , ai2 , ai3 ,... sao cho aik  ik .

Gợi ý. Bằng phản chứng, chứng minh rằng có vô số số hạng của dãy mà ai  i .

Bài 9.

 x 
Cho dãy  xn  thỏa mãn điều kiện xm n  xn  xm , m, n . Chứng minh rằng dãy  n  hội tụ.
 n 

xm 
Gợi ý. Ta thấy rằng tồn tại   0 sao cho   0 thì tồn tại m mà    .
m 2

xn xm m x
Biểu diễn n  qm  r , 0  r  m 1 để có xn  qxm  xr     r .
n m qm  r r

xn    qm  r xr  x x
Do đó            r . Đánh giá r dựa trên điều kiện đã cho để
n   2 m r 2 r r
x
chứng minh rằng   n     là xong.
n

Bài 10.

Cho dãy số an  không tuần hoàn gồm các phần tử là 0,1, 2 và xét các dãy bn  , cn  tương
ứng như sau

bn  0 nếu an  0 , bn  1 nếu an  1 hoặc an  2 .

cn  0 nếu an  2 , cn  1 nếu an  0 hoặc an  1 .

Chứng minh rằng ít nhất một trong hai dãy này không tuần hoàn.

Gợi ý. Chỉ cần chứng minh an  bn  cn , n  0,1, 2,...

Chú ý rằng hai dãy tuần hoàn thì tổng của chúng cũng phải tuần hoàn.

84
Bài 11. (VMO 1990, bảng A và B)

Cho dãy số ( xn ), n  * và x1  1 được xác định bởi hệ thức

xn  3  3xn2
xn1  , n  1, 2,3,...
2

a) Cần thêm điều kiện gì đối với x1 để dãy này gồm toàn số dương?

b) Dãy số này có tuần hoàn không? Vì sao?

Gợi ý.

3
a) Chứng minh bằng quy nạp rằng điều kiện cần tìm là 0  x1  .
2

 
b) Do xn  1, n nên có thể đặt xn  sin an ,   an  rồi xem xét tính chất của dãy an  .
2 2

Bài 12. (VMO 1994, bảng B)

Cho số thực a và dãy số  xn  xác định bởi

 x0  a
 , n  1, 2,3,...

 xn  3 6 xn1  6 sin xn1

Chứng minh rằng dãy số  xn  có giới hạn hữu hạn khi n dần tới vô cực và tìm giới hạn đó.

x3
Gợi ý. Với a  0 thì giới hạn của dãy là 0. Với a  0 thì từ bất đẳng thức sin x  x  , x  0 ,
6
ta có xn  xn1 , tức là dãy đã cho giảm, bị chặn dưới nên có giới hạn. Trường hợp a  0 thì đổi
dấu của dãy và chứng minh tương tự.

Bài 13.

Tìm điều kiện của a để các dãy số xác định như sau hội tụ:

a) u1  a, un1  un2  un  1, n  1, 2, 3,...

un2  a
b) u1  a  0, un1  , n  1, 2,3,...
2

85
Gợi ý.

a) Tiến hành khảo sát hàm số y  f ( x) và nghiệm của phương trình f ( x )  x để suy ra điều
kiện cần tìm là a  [0;1] .

x2  a
b) Hàm số tương ứng là f ( x)  có đạo hàm là f  ( x)  x và các số hạng của dãy đều
2
a2  a
dương nên hàm này đồng biến. Từ đó, chỉ cần so sánh a với là suy ra điều kiện cần tìm.
2

Bài 14.

Cho dãy số an  thỏa mãn a1  (0,1) và an 1  an  an2 , n  1, 2,3,...

Chứng minh rằng lim nan  1 .

Gợi ý.

Bằng quy nạp, ta chứng minh được: an  , n .


1
n 1

Ta cũng có: an 1  an  an2  0, n , suy ra: dãy an  giảm, đồng thời nó bị chặn dưới nên có giới

hạn, dễ thấy giới hạn đó chính là 0. Đặt cn 


1
.
an

cn 1  cn a a
 lim n n 1  lim 2 n  lim 1.
a2 1
(n  1)  n an (1  an ) 1  an
Ta thấy: lim
an .an 1

 1  lim n.an  1 .
cn
Theo định lí trung bình Cesaro, ta được lim
n

Bài 15.

Cho dãy số  xn  được xác định như sau:

 x1  a  2

 3 2 xn2  2  2
 xn 1  , n  1, 2,3,...
 2 xn  2 xn
2
 2

Xét tính hội tụ của dãy sau tùy theo giá trị của a .

86
Gợi ý.

3 2x2  2  2
Xét f ( x)  , x  2 . Ta có x1  a, xn1  f ( xn ), n  1, 2,3,...
2 x  2 x2  2

Ta thấy phơng trình f ( x )  x chỉ có hai nghiệm là x  1, x  7 .

Xét dấu của f ( x)  x trên ( , 7), (1,  ) , ta có f ( x )  x, x  7 và f ( x )  x, x  1 .

Tiếp tục nhận xét phương trình f ( x)  1 chỉ có nghiệm x  1 nên f ( x )  1 không đổi dấu trên
từng khoảng ( 1,1) và (1,  ) ; ta chứng minh được: f ( x)  1, x  1 .

Tương tự, xét phương trình f ( x)  7 , ta được: f ( x)  7, x  1 .

Từ đó suy ra:

- Nếu a  1 thì dãy giảm và bị chặn dưới nên nó hội tụ, lim xn  1 .

- Nếu a  1, a  2 thì dãy tăng và bị chặn trên nên cũng hội tụ, lim xn  7 .
Bài 16.

, f :  \     \   và dãy un  thỏa


ax  b  d  a
Cho a, b, c, d   . Xét hàm số f ( x ) 
cx  d  c  c 

u0  k  , un 1  f (un ), n  0,1, 2,...

a) Chứng minh rằng f ( x ) là một song ánh và dãy un  đã cho xác định khi và chỉ khi
d
k  vn , n , trong đó vn  được xác định bởi v0  , vn 1  f 1 (vn ), n  0,1, 2,... (lưu ý rằng dãy
c
này có thể không xác định từ một số thứ tự nào đấy).

b) Đặt   (d  a )2  4bc .

Biện luận theo  sự hội tụ của dãy un  .

Gợi ý.

ax  b b  dy   d     a  
a) Ta có y  x , ( x, y )    \       \    nên f ( x ) là một song ánh.
cx  d cy  a   c    c 
Dãy đã cho xác định khi và chỉ khi u0  k  v0 , uk xác định và khác v0 với mọi k hay k  vn , n .

87
a  b
b) Ta thấy nếu dãy đã cho có giới hạn là  thì    c 2  ( d  a)  b  0 (*) .Khi đó :
c  d

- Nếu   (d  a )2  4bc  0 thì dãy này phân kì.

- Nếu   (d  a )2  4bc  0 thì (*) có hai nghiệm phân biệt, đặt là    .

Ta lại xét các trường hợp :

+ Nếu u0    un  a, n . Dãy đã cho là dãy hằng.

un 1   c  d un  
+ Nếu u0   , ta xét dãy số : U n 1     .U n . Suy ra :
un 1   c  d un  
.

un 1   c  d un   c  d
U n 1     .U n  U n   n .U 0 với    1.
un 1   c  d un   c  d
.

   1  un  
Do đó : 
   1  un  

Trường hợp   1 thì un   nếu u0   và un  phân kì nếu u0   .

ad
- Nếu   (d  a )2  4bc  0 , đặt   là nghiệm duy nhất của (*).
2c

+ Nếu u0    un  a, n .

+ Nếu u0   . Ta chứng minh được dãy Vn    khi n   .


un  
1

Từ đó suy ra : lim un   .

Bài 17.

Dãy số an  được xác định bởi a1  0 , a2  0 và an1  an  an1 .


Chứng minh rằng dãy số an  hội tụ và tìm giới hạn của dãy số đó.

Xét dãy số M n  max an , an1 , 4 .


Gợi ý.

+ Nếu M n  4 thì an , an 1  4 , suy ra an 2  4 , từ đó M n 1  4 .


+ Nếu M n  an 1 thì an 1  an , an 1  4 . Khi đó

88
an1  an1  an  an 1  an 1  an1 suy ra an 2  an  an1  an  an 1  an 1 .
Do đó, M n1  max an1 , an 2 , 4  an1 .
+ Nếu M n  an thì an  an 1 , an  4 . Khi đó an  2  an  an 1  2 an  an .
Suy ra M n 1  an  M n .
Vậy trong mọi trường hợp thì M n 1  M n , tức là dãy M n  là dãy số giảm. Do M n  bị chặn d-
ưới bởi 4 nên dãy này có giới hạn. Ta chứng minh giới hạn này bằng 4. Thực vậy, giả sử giới hạn
là M  4 . Khi đó với mọi   0 , tồn tại N sao cho với mọi n  N thì M    M n  M   .
Chọn n  N sao cho M n  2  an  2 (theo lập luận trên và do M  4 thì tồn tại chỉ số n như vậy).
Ta có
M    M n 2  an 2  an  an1  2 M  
 M  M  4     2M  4     0
Mâu thuẫn vì M  4 và  có thể chọn nhỏ tuỳ ý.
Do đó, lim M n  4 suy ra dãy đã cho cũng hội tụ tại 4.
Bài 18. (Việt Nam TST 1990)

Cho trước bốn số thực dương a, b, A, B . Xét dãy số  xn  xác định như sau

x1  a, x2  b, xn  2  A 3 xn21  B 3 xn2 , n  1, 2,3,...

Chứng minh rằng tồn tại giới hạn lim xn và tìm giới hạn đó.
n 

Gợi ý.

Xét dãy số phụ bn  max{an , an 1 , ( A  B )3 } .

của dãy này và dãy  xn  đã cho có cùng giới hạn là ( A  B )3 .


Ta chứng minh được dãy này giảm và bị chặn dưới nên có giới hạn. Sau đó chứng minh giới hạn

Bài 19. (VMO 1992)

Cho ba số thực dương a, b, c và các dãy số ak  , bk  , ck  , k  0,1, 2,... được xác định như sau:

1) a0  a, b0  b, c0  c .

2) ak 1  ak  , bk 1  bk  , ck 1  ck  ,k  0
2 2 2
bk  ck ck  ak ak  bk

Chứng minh các dãy ak  , bk  , ck  này dần tới vô cực khi n tiến tới vô cực.
89
Gợi ý.
Với mỗi k  0 , đặt M  max{ak , bk , ck } và m  min{ak , bk , ck } , dễ thấy các dãy này đều dương.

Ta sẽ chứng minh rằng: lim M k  , lim  p    . Thật vậy:


Mk
mk
Xét M k , từ giả thiết, ta có:
 ak 
ak21  bk21  ck21  ak2  bk2  ck2  2      ak  bk  ck  6
bk ck
 bk  ck ck  ak ak  bk 
2 2 2

ak2  bk2  ck2


Bằng quy nạp, suy ra: a  b  c  6k , k  M 
2
k
2
k
2
k
2
k  2k  lim M k   .
3

, ta chứng minh được: mk 1  mk  , M k 1  M k  , k . Suy ra:


Mk 1 1
Xét
mk Mk mk

 1   1 
M k 1.mk   M k   mk  M k  mk    M k .mk 1  
M k 1 M k
 mk   Mk 
.
mk 1 mk

 1 , suy ra dãy k này hội tụ, tức là tồn tại giới hạn p của k .
Mk M M
Hơn nữa rõ ràng:
mk mk mk

Từ đó suy ra: lim mk   và suy ra đpcm.

Bài 20.

Cho dãy số dương an  thỏa mãn: a1  0, anp1  a1  a1  ...  an , n  1 với 0  p  2 .


Chứng minh rằng tồn tại c  0 sao cho an  nc, n .
Gợi ý.

Từ giả thiết, suy ra: an  a1 , n hay an  (n  2)a1  a1 , n .


1 1
p p

Do đó: lim an   . Suy ra, tồn tại N 0   sao cho với mọi n  N 0 thì: an  1 .

Đặt c  min{ , a1 , 2 ,..., 0 }  n  c  0, n  N 0 .


1 a aN a
4 2 N0 n
Ta sẽ chứng minh an  nc, n  N 0 bằng quy nạp. Thật vậy:
Với n  N 0 , nhận xét trên hiển nhiên đúng.
Giả sử nhận xét đúng đến n  N1  N 0 , ta có

c  ( N1  1)c 1   ( N1  1)c 
N1 ( N1  1)
aN2 1 1  anp1  a1  a2  ...aN1  (1  2  ...  N1 ).c 
N 2

2 2

90
 aN1 1  ( N1  1)c   c , tức là nhận xét trên cũng đúng với n  N1  1 .
aN1 1
N1  1
Theo nguyên lí quy nạp, nhận xét được chứng minh. Từ đây ta cũng suy ra đpcm.

Bài 21.
Khảo sát sự hội tụ của dãy số cho bởi công thức sau
u0  a  , un1  3 7un  6, n  0,1, 2,...

Gợi ý.
Đặt f ( x )  3 7 x  6 , dễ thấy f ( x ) đồng biến trên  .
Ta thấy phương trình f ( x )  x có 3 nghiệm phân biệt là x  3, x  1, x  2 .
Xét dấu của f ( x ) trên từng khoảng (, 3), (3,1), (1, 2), (2, ) , ta được:
a  1  lim un  3

a  1  lim un  1
a  1  lim u  2
 n

Cho   (0, 2) . Tính giới hạn của dãy sau theo các giá trị u0 , u1 cho trước
Bài 22.

un  2   un 1  (1   )un , n  0,1, 2,...

Gợi ý.
Ta có: un 1  un  (  1)(un  un 1 )  ...  (  1) n (u1  u0 ) .

Suy ra: un 1  u0   (uk  uk 1 )   (  1) k (u1  u0 )  (u1  u0 ) (  1) k  (u1  u0 ).


n 1 n n
1  (  1)n
k 1 k 0 k 0 1  (  1)
Từ   (0, 2) , ta được: 1    1  1    1  1  lim  (  1)n   0 .
 1  (  1)n  u1  u0
Do đó: lim(un 1  u0 )  lim (u1  u0 ). 
 1  (  1)  2  
.

u u (1   )u0  u1
Vậy lim un  1 0  u0 
2  2 
.

Bài 23. (Việt Nam TST 1991)

Cho dãy thực dương  xn  được xác định bởi


x1  1, x2  9, x3  9, x4  1, xn 4  4 xn .xn1.xn 2 .xn 3 , n  1 .
Chứng minh dãy này có giới hạn hữu hạn. Tìm giới hạn đó.

91
Xét dãy  yn  yn  log 3 xn , n , tức là
Gợi ý.

yn  yn 1  yn  2  yn 3
y1  0, y2  2, y3  2, y4  0, yn  4  , n  1.
4
Ta sẽ chứng minh dãy  yn  này hội tụ. Thật vậy:

Từ công thức tổng quát này, suy ra


4 yn  4  yn  yn 1  yn  2  yn 3  4 yn 4  3 yn 3  2 yn  2  yn 1  4 yn 3  3 yn  2  2 yn 1  yn

Bằng quy nạp, ta chứng minh được


4 yn  4  3 yn 3  2 yn 2  yn 1  4 y3  3 y2  2 y1  y0  4.0  3.2  2.2  1.0  10

Suy ra nếu dãy  yn  có giới hạn là  thì 10  10    1 .

Ta cũng dùng dãy phụ để chứng minh dãy này có giới hạn.
Xét dãy un  max{ yn , yn 1 , yn  2 , yn 3}, vn  min{ yn , yn 1 , yn  2 , yn 3}, n .

Bằng quy nạp, dễ thấy yn  9, n .

Ta chứng minh được dãy un  giảm và bị chặn dưới, dãy vn  tăng và bị chặn trên nên cả hai
dãy này đều hội tụ tại một điểm. Suy ra dãy  yn  hội tụ tại   1 .

Vậy dãy  xn  hội tụ và lim xn  3 .

92
F – CÁC BÀI TOÁN TỰ GIẢI.

Bài 1.

n n 1
Trên hypebol xy  1 , ta lần lượt lấy các điểm An , Bn với các hoành độ tương ứng là , .
n 1 n

Kí hiệu M n là tâm của đường tròn đi qua An , Bn và các đỉnh của hypebol. Hãy tìm giới hạn của
dãy M n  khi n tiến tới vô cực.

Bài 2.

1 a
Giả sử xk 1  2 xk  2  ,  a  0, x0  0 .
2  xk 

a. Chứng minh rằng dãy số  xn  có giới hạn hữu hạn. Tìm giới hạn đó.

b. Kí hiệu zn là hiệu giữa xk và gi

ới hạn tương ứng của dãy với giả thiết rằng z1  0 . Chứng minh rằng với mọi k  1 thì ta có các
bất đẳng thức sau
2 1
zk  0, zk 1  zk , z k 1  3 zk
3 a
Bài 3.

Giả sử x1 , x2 , x3 ,... là tập hợp tất cả các nghiệm dương của phương tan x  x được xếp theo thứ
tự tăng dần. Tính giới hạn của dãy yn  xn 1  xn , n  1, 2,3,...

Bài 4.

Giả sử dãy an  là dãy các số dương và dãy  xn  xác định bằng cách sau

x1  1, x2  2, xn2  xn  an xn 1 , n  1 .

Chứng minh rằng giữa các phần tử của dãy  xn  có vô hạn các số nguyên dương và nguyên âm.

Bài 5.

Giả sử p là số nguyên tố và a, b sao cho a 2  ab  b 2  0(mod p) . Xét dãy số nguyên dương sau

v0  a, v1  b, vn 1  vn  vn1 , n  1, 2,3,...
93
Chứng minh rằng số dư vn mod p tuần hoàn và có chu kì không phụ thuộc vào a, b .

Bài 6.

Giả sử an  là dãy hữu hạn các số thực 1  n  N . Ta gọi ak là một số được đánh dấu nếu như
có ít nhất một trong các số ak , ak  ak 1 ,..., ak  ak 1  ...  ak  2011 là không âm (nếu n  N thì
coi an  0 ). Chứng minh rằng tổng tất cả các số được đánh dấu là không âm.

Bài 7.

a0  1, a1  p, a2  p 2  q,

Cho dãy số sau  .
an3  pan2  qan1  an , n  0

Biết an  0, n và am an  am1an1 , m  n . Chứng minh rằng đa thức P ( x )  x 3  px 2  qx 1


có đúng ba nghiệm thực.

Bài 8.

Cho dãy số nguyên dương an  thỏa mãn điều kiện 0  an1  an  an , n  1 .

ak
Chứng minh rằng với mọi x, y mà 0  x  y  1 thì tồn tại k , l  * sao cho x   y.
al

Bài 9.

Cho số thực m  0 và dãy số an  thỏa mãn

 a2 
, an1   2n  2 an , n  1, 2,3,...
1
a0  1, a1  m 
m  an1 

n
1
Chứng minh rằng tổng yn   bị chặn.
i 1 ai

Bài 10.

Cho hai dãy số an  , bn  thỏa mãn điều kiện

an  an1 an  an2   bn  bn1 bn  bn2   0, n  2

Chứng minh rằng tồn tại số nguyên dương k sao cho ak  ak  2011 .

94
Bài 11.

 1  u1  a
Chứng minh rằng với mọi a  1; e e  thì dãy số  hội tụ.
  un1  a un , n  1, 2,3,...

Bài 12.

Cho hai dãy số thực dương an  , bn  thỏa mãn điều kiện

(i) a0  1  a1 , an (bn1  bn1 )  an1bn1  an1bn 1 , n  1

n
(ii) b  n
i 1
i n , n  1.

Tìm công thức tổng quát của an  .

Bài 13.

Cho dãy số nguyên dương bn  và dãy số an  với a1 cố định được xác định như sau

an 1  an bn 1, n  1 .

Tìm tất cả các số nguyên dương m lớn hơn 2 thỏa mãn điều kiện:

(i) Dãy số an mod m tuần hoàn.

(ii) Tồn tại các số nguyên dương q, u, v sao cho 2  q  m 1 và dãy bu bt mod q tuần hoàn.

* Tài liệu tham khảo.

[1] Đề thi chọn đội tuyển các trường, đề thi HSG các tỉnh, thành phố năm học 2011 – 2012.

[2] Trần Nam Dũng, Tài liệu giáo khoa chuyên Toán 11, NXB Giáo dục, 2010.

[3] Trần Lưu Cường, Toán Olympic cho Sinh viên, NXB Giáo dục, 1998.

[4] Tủ sách tạp chí THTT, Các bài toán thi Olympic Toán THPT, NXB Giáo dục, 2007.

[5] Tuyển tập 30 năm tạp chí THTT, NXB Giáo dục, 1996.

[6] Các diễn đàn Toán học http://mathscope.org, http://mathlinks.ro.

95

You might also like